56666

Ace your homework & exams now with Quizwiz!

A 42-year-old male presents to triage with a chief complaint of "something in his right eye." He was cutting tree limbs and thinks something went into his eye. No past medical history, no allergies, no medications. On exam, his right eye is reddened and tearing. Pain is 4/10.

ESI level 4: One resource. The only resource this patient will require is irrigation of his eyes. A slit lamp exam is not considered a resource but is part of the physical exam.

EMS arrives in the ED with a 57-year-old female with multiple sclerosis. She is bedridden, and her family provides care in the home. The family called 911 because her Foley catheter came out this morning. No other complaints. Vital signs are within normal range, currently on antibiotics for a UTI.

ESI level 4: One resource. The patient was brought to the emergency department for a new Foley catheter—one resource. There are no other changes in her condition, and she is already on antibiotics for a UTI, so no further evaluation is needed.

"I just need another prescription for pain medication. I was here 10 days ago and ran out," a 27-year-old male tells you. "I hurt my back at work, and it's still bothering me." Denies numbness, tingling, or bladder or bowel issues. Vital signs are within normal limits. Pain 10+/10.

ESI level 5: No resources. No resources are required. Following a physical exam, this patient will be sent home with appropriate discharge instructions and a prescription if indicated.

Which of the following groups of patients tends to be undertriaged? Select 1 answer. 18-25 year olds 30-40 year olds 50-60 year olds E. 65-80 year olds

E

*32. Match the following actions with the appropriate responsibility regarding the delegation of nursing interventions: 1. UAP's statement, "I'll notify you of Mr. Smith's BP as soon as I take it so you can decide whether he gets his medication." 2. RN discusses the way the LPN handled a patient complaint 3. RN initially observes a newly hired LPN is changing a simple dressing a. Appropriate supervision b. Evaluation and reassessment c. Effective communication

1. C. Effective communication 2. B. Evaluation and reassessment 3. A. appropriate supervision

1. The nurse is assigned to care for four clients. In planning client rounds, which client should the nurse assess first? a. A client scheduled for a chest x-ray b. A client requiring daily dressing changes c. A postoperative client preparing for discharge d. A client receiving nasal oxygen who had difficulty breathing during the previous shift.

1. D- Airway is always the highest priority, and the nurse would attend to the client who has been experiencing an airway problem first. The clients described in option 1, 2, and 3 have needs that would be identified as intermediate priorities.

10. The nurse should use which guideline(s) to plan delegation and assignment-making activities? Select all that apply. a. Ensuring client safety b. Requests from the staff c. The clustering of the rooms on the unit d. The number of anticipated client discharges e. Client needs and workers' needs and abilities

10. A, E- There are guidelines that the nurse should use when delegating and planning assignments. These include the following: ensure client safety; be aware of individual variations in work abilities; determine which tasks can be delegated and to whom; match the task to the delegate on the basis of the nurse practice act and appropriate position descriptions; provide directions that are clear, concise, accurate and complete; validate the delegate's understanding of the directions; communicate a feeling of confidence to the delegate, and provide feedback promptly after the task is performed; and maintain continuity of care as much as possible when assigning client care. Staff requests, convenience as in clustering client rooms, and anticipated changes in unit census are not specific guidelines to use when delegating and planning assignments.

2. The nurse employed in an emergency department is assigned to triage clients coming to the emergency department for treatment on the evening shift. The nurse should assign priority to which client? a. A client complaining of muscle aches, a headache, and malaise b. A client who twisted her ankle when she fell while rollerblading c. A client with a minor laceration on the index finger sustained while cutting an eggplant d. A client with chest pain who states that he just are pizza that was made with a very spicy sauce.

2. D- In an emergency department, triage involves brief client assessment to classify clients according to their need for care and includes establishing priorities of care. The type of illness or injury, the severity of the problem, and the resources available govern the process. Clients with trauma, chest pain, severe respiratory distress or cardiac arrest, limb amputation, and acute neurological deficits, or who have sustained chemical splashes to the eyes, are classified as emergent and are the number 1 priority. Clients with conditions such as a simple fracture, asthma without respiratory distress, fever, hypertension, abdominal pain, or a renal stone have urgent needs and are classified as a number 2 priority. Clients with conditions such as a minor laceration, sprain, or cold symptoms are classified as nonurgent and are a number 3 priority.

3. A nursing graduate is attending an agency orientation regarding the nursing model of practice implanted in the health care facility. The nurse is told that the model is a team nursing approach. The nurse understands that planning care delivery will be based on which characteristic of this type of nursing model of practice. a. A task approach method is used to provide care to clients b. Managed care concepts and tools are used in providing client care. c. A single registered nurse is responsible for providing care to a group of clients d. A registered nurse leads nursing personnel in providing care to a group of clients.

3. D- In team nursing, nursing personnel are led by a registered nurse leader in providing care to a group of clients. Option 1 identifies functional nursing. Option 2 identifies a component of case management. Option 3 identifies primary nursing (relationship-based practice).

4. A nurse has received the assignment for the day shift. After making initial rounds and checking all of the assigned clients, which client should the nurse plan to care for first? a. A client who is ambulatory b. A client scheduled for physical therapy at 1 pm c. A client with a fever who is diaphoretic and restless d. A postoperative client who has just received pain medication

4. C- The nurse should plan to care for the client who has a fever and is diaphoretic and restless first because this client's needs are the priority. The client who is ambulatory and the client scheduled for physical therapy later in the day do not have priority needs related to care. Waiting for pain medication to take effect before providing care to the postoperative client is best.

5. The nurse is giving a bed bath to an assigned client when an unlicensed assistive personnel (UAP) enters the client's room and tells the nurse that another assigned client is in pain and needs pain medication. Which is the most appropriate nursing action? a. Finish the bed bath and then administer the pain medication to the other client b. Ask the UAP to find out when the last pain medication was given to the client c. Ask the UAP to tell the client in pain that medication will be administered as soon as the bed bath is complete. d. Cover the client, raise the side rails, tell the client that you will return shortly, and administer the pain medication to the other client.

5. D- The nurse is responsible for the care provided to assigned clients. The appropriate action in this situation is to provide safety to the client who is receiving the bed bath and prepare to administer the pain medication. Options 1 and 3 delay the administration of medication to the client in pain. Option 2 is not responsibility of the UAP.

6. The nurse manager has implemented a change in the method of the nursing delivery system from functional to team nursing. An unlicensed assistive personnel (UAP) is resistant to the change and is not taking an active part in facilitating the process of change. Which is the best approach in dealing with the UAP? a. Ignore the resistance b. Exert coercion on the UAP c. Provide a positive reward system for the UAP d. Confront the UAP to encourage verbalization of feelings regarding the change

6. D- Confrontation is an important strategy to meet resistance head on. Face-to face meetings to confront the issue at hand will allow verbalization of feelings, identification of problems and issues, and development of strategies to solve the problem. Option 1 will not address the problem. Option 2 may produce additional resistance. Option 3 may provide a temporary solution to the resistance, but will not address the concern specifically

7. The registered nurse is planning the client assignments for the day. Which is the most appropriate assignment for an unlicensed assistive personnel (UAP)? a. A client requiring a colostomy irrigation b. A client receiving continuous feedings c. A client who requires urine specimen collections d. A client with difficulty swallowing food and fluids

7. C- The nurse must determine the most appropriate assignment based on the skills of the staff member and the needs of the client. In this case, the most appropriate assignment for the UAP would be to care for the client who requires urine specimen collections. The UAP is skilled in this procedure. Colostomy irrigations and tube feedings are not performed by unlicensed personnel. The client with difficulty swallowing food and fluids is at risk for aspiration

8. A new unit nurse manager is holding her first staff meeting. The manager greets the staff and comments that she has been employed to bring about quality improvement. The manager provides a plan that she developed and a list of tasks and activities for which each staff member must volunteer to perform. In addition, she instructs staff members to report any problems directly to her. What type of leader and manager approach do the new manager's characteristics suggest? a. Autocratic b. Situational c. Democratic d. Laissez-faire

8. A- The autocratic leader is focused, maintains strong control, makes decisions, and addresses all problems. The autocrat dominates the group and commands rather than seeks suggestions or input. In this situation, the manager addresses a problem (quality improvement) with the staff, designs a plan without input, and wants all problems reported directly back to her. A situational leader will use a combination of styles, depending on the needs of the group and the tasks to be achieved. The situational leader would work with the group to validate that the information that the leader gained as a new employee was accurate and that a problem existed then the leader would take the time to get to know the group determine which approach to change (if needed) would work best according to the needs of the group and the nature and substance of the change that was required. A democratic leader is participative and would likely meet with each staff person individually to determine the staff member's perception of the problem. The democratic leader would also speak with the staff about any issues and ask the staff for input with developing a plan. A laissez-faire leader is passive and nondirective. The laissez-faire leader would state what the problem was and inform the staff that the staff needed to come up with a plan to "fix it."

9. The nurse employed in a long-term care facility is planning assignments for the clients on a nursing unit. The nurse needs to assign four clients and has a licensed practical (vocational) nurse and three unlicensed assistive personnel (UAP) on a nursing team. Which client would the nurse most appropriately assign to the licensed practical (vocational) nurse? a. A client who requires a bed bath b. An older client requiring frequent ambulation c. A client who requires hourly vital sign measurements d. A client requiring abdominal wound irrigations and dressing changes every 3 hours

9. D- When delegating nursing assignments, the nurse needs to consider the skills and educational level of the nursing staff. Giving a bed bath, assisting with frequent ambulation, and taking vital signs can be provided most appropriately by the unlicensed assistive personnel (UAP). The licensed practical (vocational) nurse is skilled in wound irrigations and dressing changes and most appropriately would be assigned to the client who needs this care

*32. Match the nursing model most utilized in the following nursing care settings? 1. Functional 2. Case Management 3. Team 4. Total a. Emergency department b. Medical surgical unit c. Critical care d. Home health

ANS: 1. Functional -- a. Emergency Dept 2. Case Management-- d. Home health 3. Team-- b. Medical-surgical unit 4. Total -- c. Critical care

A mother brings in a 4-year-old child to triage. The child is wrapped in a blanket. When the triage nurse unwraps the child, she finds that the child is gray, with a Glasgow Coma Scale (GCS) score of 3. Breathing is about a 6 to 8 and capillary refill is greater than 6 seconds. The mother explains that the child has not eaten or had fluids for 2 days, and has had diarrhea. What ESI level would you assign to this patient and why? Select 1 answer. A. ESI 1: The child requires emergent resuscitation B. ESI 2: The child is ill and falls under the acuity scale C. ESI 3: The child requires several resources D. ESI 4: The child requires a pediatric electrolyte solution and observation

A

The ESI Levels 3, 4 and 5 are determined by the number of resources anticipated to be necessary during the patient's stay in the emergency department (ED). Which of the following is considered a resource? A. Intramuscular ceftriaxone for gonorrhea treatment B. Primary care physician phone notification C. Insertion of an intravenous catheter D. Physical exam

A

Which of the following patients meets criteria for an ESI Level 2? Select 1 answer. A. A 36-year-old man presents with midepigastric pain that has migrated to the right lower quadrant over the last several hours B. A 22-year-old college student was at a party last evening. She doesn't remember what happened but she awoke in a strange bed with someone she does not know and was wearing different clothes; she is tearful C. A 45-year-old man was playing squash when he had sudden onset of pain in his left calf; he has no other complaints and is comfortable as long as he does not walk D. A 36-year-old man was found lying on the floor of a bathroom in a fast food restaurant with a needle in his arm; he has a respiratory rate of 4 breaths per minute

A

*25. A _____________ is a measurable condition that results from interventions that can be either positive or negative in nature.

ANS: *patient outcome* Patient outcome is a measurable condition that results from interventions by the health care team; a change in a person's health after treatment; outcomes may be positive such as improved mobility or improved lab values or negative such as infections, falls, or death. p. 366

After a vasectomy, what teaching should be included in the discharge teaching? A. "You will want to use an alternative form of contraception for 6 weeks." B. "You may lose some secondary sexual characteristics after this surgery." C. "You may have erectile dysfunction for several months after this surgery." D. "You will be uncomfortable, but you may safely have sexual intercourse today."

A. "You will want to use an alternative form of contraception for 6 weeks."

*27. The active process of directing, guiding, and influencing the outcome of an individual's performance of an activity or task is referred to as ___________________.

ANS: *supervision* Supervision is the active process of directing, guiding, and influencing the outcome of an individual's performance of an activity or task. p. 352

3. Nurses on a unit provide personal hygiene, administer medications, educate the patient and family about treatments, and provide emotional support. These nurses provide patient care based on which nursing delivery system? a. Total patient care b. Partnership nursing c. Team nursing d. Functional nursing

ANS: A In total patient care nurses provide all aspects of patient care. p. 370

*26. What factors will be focal considerations when a facility begins developing its own set of clinical pathways? (Select all that apply.) a. The facility's most commonly treated diagnoses b. The most costly diagnoses treated by the facility's staff c. Accepted standards of specialized nursing care d. Dietary and physical therapy are viewed as principal contributors e. Physician input as a primary source of decision making

ANS: A, B, C Clinical pathways most often are developed for the health care facility's most common or costly diagnoses. Clinical pathways should also be based on accepted standards of practice as recommended by specialty nursing organizations. The success of clinical pathway development and implementation depends on input and support from all disciplines, including physicians, involved in using the pathway and caring for the patient. pp. 374-375

A 71-year-old patient with a diagnosis of benign prostatic hyperplasia (BPH) has been scheduled for a contact laser technique. What is the primary goal of this intervention? A. Resumption of normal urinary drainage B. Maintenance of normal sexual functioning C. Prevention of acute or chronic renal failure D. Prevention of fluid and electrolyte imbalances

A. Resumption of normal urinary drainage

A 33-year-old patient noticed a painless lump in his scrotum on self-examination of his testicles and a feeling of heaviness. The nurse should first teach him about what diagnostic test? A. Ultrasound B. Cremasteric reflex C. Doppler ultrasound D. Transillumination with a flashlight

A. Ultrasound

*30. When considering supervisory responsibilities, the registered nurse (RN) must recognize that: a. When two RNs work together to move a client, neither nurse is in a supervisory position. b. Supervision is required only when staff is new to the unit, the patient, or the task. c. All supervision is done with the RN being immediately available to the staff. d. A charge nurse is actually supervising only the LPNs and UAPs on the unit.

ANS: A One RN is working with another RN in a collegial relationship, and neither RN is in the position of supervising the other. Each RN is responsible and accountable for his or her own practice. However, the RN in a supervisory or management position (e.g., team leader, charge nurse, nurse manager), as defined by the health care organization, will be in a position to supervise other RNs. Appropriate supervision is a responsibility of any delegation of task, not just when the staff is unfamiliar with the patient, the task, or the unit. Supervision may be categorized as on-site, in which the nurse is physically present or immediately available while the activity is being performed, or off-site, in which the nurse has the ability to provide direction through various means of written, verbal, and electronic communication.REF: pp. 361-362

1. The task of completing and signing the initial assessment on a newly admitted patient who is about to undergo minimally invasive procedures on an outpatient basis can be delegated to: A. The registered nurse (RN). B. The licensed practical/vocational nurse (LPN/LVN). C. Unlicensed assistive personnel (UAP). D. All levels of staff, because the information is about the past and cannot change.

ANS: A Only the RN can perform and sign the admission assessment, although some components such as monitoring vital signs may be delegated. p. 357

16. A nurse groups patients with criteria such as: "high risk for falls", "infection protocols", and "special communication needs" to determine the mix and number of staff needed on a telemetry unit. The nurse is using: A. a patient classification system to determine safe staffing levels. B. diagnostic-related groups for Medicare billing. C. case management to coordinate care. D. clinical pathways to determine care.

ANS: A Patient classification systems group patients according to care needs to determine safe staffing levels. p. 367

3. Which task is most likely to be considered in a state's practice act as appropriate to delegate to an LPN/LVN if the patient 's condition is stable and competence in the task has been established? A. Administer an enema for an elective surgery patient. B. Administer an antiarrhythmic medication IV while interpreting the patient's rhythm on the cardiac monitor. C. Develop a plan of care for a stable patient admitted for observation after a head injury. D. Teach a patient how to instill eye drops for glaucoma.

ANS: A The RN who is delegating must consider the following: (1) the delegatee's current workload and the complexity of the task, (2) whether the staff member is familiar with the patient population and with the task to be performed, and (3) whether the RN is able to provide the appropriate level of supervision. The delegation decision-making tree would also support the delegation of this task. pp. 357-358, Box 20-3

19. A nurse is concerned about the risk of delegating tasks to licensed practical nurses and unlicensed assistive personnel. What is the best way for the nurse to determine competency of an inexperienced delegatee? A. Actually observe the delegatee perform the assigned task. B. Ask the delegatee how many times he/she has performed the task. C. Ask the patient if the care provided was satisfactory. D. Ask other nurses if they feel the delegatee is competent.

ANS: A The best way for the nurse to determine the competency of LPNs or UAPs is to observe them perform the task. p. 361

16. The RN instructs the LPN to "Give an enema to the patient in room 327 who is being discharged but is complaining of being constipated. Then be sure to document on the medication administration record when given." Which of the five rights was missing in this situation? The right of: A. direction and communication. B. task. C. person. D. circumstances.

ANS: A The directions were not clear. The RN did not specify which type of enema to give and what outcome to expect. And the RN gave no instructions related to reporting back. p. 360

10. The nurse manager is planning staffing levels and realizes that the first step is to: a. know the intensity of care needed by patients according to physical and psychosocial factors. b. examine the educational level of the staff. c. assess the skill level of caregivers. d. review the budget to determine the financial consequences of past staffing patterns.

ANS: A The nurse manager must determine the number and mix of health care providers according to the wide range of care requirements of individual patients. p. 367

4. A patient is admitted with hypotension, shortness of breath, flushing, and hives. All levels of staff have been trained to assess vital signs. Given budget restrictions and proper delegation rules, to which care provider would the RN delegate the task of obtaining the initial blood pressure reading? A. RN B. LPN/LVN C. Unlicensed assistive personnel (UAP) D. Use the blood pressure obtained in the ambulance, because it was assessed via electronic monitoring.

ANS: A The patient's condition is not stable; therefore, the skills of an RN are required. pp. 357-358, Box 20-3

21. Which statement made by an RN regarding delegation indicates the need for additional teaching? *select all that apply* A. Unlicensed assistive personnel (UAP) can assess vital signs during the first 5 minutes for a patient who is receiving a blood transfusion because a reaction at this time is unlikely. B. An LPN/LVN can administer a PPD (tuberculin skin test) if there is no history of a positive PPD. C. When dopamine is ordered continuously, the LPN/LVN can administer dopamine at a low dose for the purpose of increasing renal perfusion. D. UAPs can transfer a patient who is being discharged home from the wheelchair to the bed if they have received training and demonstrated competency. E. Responsibility can be delegated to the UAP, but the delegator retains accountability.

ANS: A, B, C The statement "UAPs can assess vital signs during the first 5 minutes for a patient who is receiving a blood transfusion because a reaction at this time is unlikely" indicates the need for further teaching because the patient is at highest risk of a reaction during the first few minutes of a blood transfusion; thus the assessment skills of an RN are required. The statement "an LPN/LVN can administer a PPD (tuberculin skin test) if there is no history of a positive PPD" indicates the need for further teaching because administration of intradermal medication requires the skill of an RN. Dopamine is a vasoactive drug that can have a profound effect on a patient's blood pressure and cardiac output; administration requires the assessment and evaluation skills of an RN. pp. 357-358, Box 20-3

23. A nurse responsible for staffing a medical-surgical unit must consider: (select all that apply) a. the patient census. b. physical layout of the unit. c. complexity of care required. d. educational level of all staff. e. task preferences of the nurses.

ANS: A, B, C, D The primary considerations for staffing a specific nursing unit are the number of patients; the level of intensity of care required by those patients (commonly referred to as patient acuity); contextual issues, such as architecture, geography of the environment, and available technology; level of preparation and experience of the staff members providing the care; and the quality of the nurses' work life. pp. 367-368

23. Which functions can be delegated only to another RN with appropriate experience and training? *select all that apply* A. Assessment of skin integrity on the third day of hospitalization B. Evaluation of patient teaching related to turning, coughing, and deep breathing exercises C. Nursing judgment related to withholding medication based on vital signs D. RNs do not delegate to other RNs, they delegate only to licensed practical nurses or unlicensed assistive personnel E. Formulation of nursing diagnosis "potential for fall"

ANS: A, B, C, E Activities like assessing skin integrity—which include the core of the nursing process and require specialized knowledge, judgment, and/or skill—can be delegated only to another RN. Activities like evaluating patient teaching—which include the core of the nursing process and require specialized knowledge, judgment, and/or skill—can be delegated only to another RN. Activities like deciding to withhold medication based on vital signs—which include the core of the nursing process and require specialized knowledge, judgment, and/or skill—can be delegated only to another RN. Activities like formulating a nursing diagnosis—which include the core of the nursing process and require specialized knowledge, judgment, and/or skill—can be delegated only to another RN. pp. 361-362

*25. Which situations demonstrate effective delegation by the RN to the LPN? *Select all that apply* A. RN asks the LPN, "Do you have any concerns about your assignment for today?" B. LPN asks, "Where will you be if I need help with the dressing I've been assigned?" C. RN adjusts the LPN's assignments when a patient's condition becomes unstable D. LPN states, "I'll do whatever is necessary to keep my patients free of pain." E. RN assumes responsibility for ambulating a patient when he reports, "I'm feeling dizzy."

ANS: A, B, C, E The RN is responsible for communicating effectively with the LPN concerning responsibilities and accountability regarding patient care. *Asking the LPN if she/he has any concerns demonstrates effective communication*. Appropriate supervision is demonstrated by the LPN's when a question about support is asked. Evaluation and reassessment is demonstrated by the RN adjusting assignments and assuming a responsibility when the patients' conditions change. The LPN's statement about doing whatever is necessary demonstrates a possible risk for actions outside the established job description, competency, and standard of care on the part of the LPN. p. 360, Box 20-4

22. Which factors would be considered in the first steps in developing an effective patient classification system? (select all that apply) a. Planned procedures b. Ethnic diversity of patients c. Clinical competency of staff d. Educational level of nurses e. Age of patients

ANS: A, B, E The first step in developing a patient classification system is to understand the intensity of care needs, which requires identifying specific patient characteristics and care requirements. p. 368

*24. Which statements reflect common nurse practice acts policies? *Select all that apply* A. Only nursing tasks can be delegated, not nursing practice B. The LPN practices professional nursing C. The RN can delegate only what is within the scope of nursing practice D. The RN must evaluate patient outcomes resulting from the delegated activity E. The LPN works under the supervision and direction of the RN

ANS: A, C, D, E The common policies reflected in nurse practice acts include: only nursing tasks can be delegated, not nursing practice, the RN can delegate only what is within the scope of nursing practice, the LPN or LVN works under the direction and supervision of the RN, the RN must evaluate patient outcomes resulting from the delegated activity. The LPN or LVN and UAP do not practice professional nursing. REF: p. 355, Box 20-1

21. While participating in a task force to proactively plan for nursing care delivery over the next 20 years, a nurse learns that dramatic changes will occur as a result of: (select all that apply.) A. the increase in the number of minimally invasive procedures being performed for disease treatment. B. care provided for patients over an extended period in acute care settings. C. the reduction in the number of nurses and other health care professionals who are available to provide care. D. the widespread illiteracy and decreased self-efficacy of the aging patient population. E. the need to focus on social and environmental influences, educational level, and individual characteristics and values of the patient. F. the devaluing of nursing as a means of improving patient outcomes.

ANS: A, C, E Invasive surgical procedures are being replaced by laparoscopic procedures. The demand for nurses and other health care professionals cannot keep pace with the increased need for health care required by the growing older population. Care will focus on the unique lifestyles and values of a diverse population. p. 376

*32. When considering nursing care what events are associated with the term "churning" and responsible for up to 70% of nursing care on a typical 20 bed medical-surgical unit (Select all that apply.) a. Seven patients have been written discharge orders by their physicians b. Five patients will have initial visits from physical therapy staff c. Twelve patients have needs that require assistance with bathing and elimination d. Five patients will be admitted for scheduled surgeries tomorrow AM e. Two patients are in the ER department with orders to be admitted as soon as a bed is available

ANS: A, D, E Fast-paced patient turnover in acute care settings; the rapid discharge and admission cycle is now referred to as "churning" and is estimated to range from 25% to 70% on a typical medical-surgical unit. The remaining options reflect standard unit care. p. 376

7. A patient is admitted with pneumonia. The case manager refers to a plan of care that specifically identifies dates when supplemental oxygen should be discontinued, positive-pressure ventilation with bronchodilators should be changed to self-administered inhalers, and antibiotics should be changed from intravenous to oral treatment, on the basis of assessment findings. This plan of care is referred to as a: a. patient classification system. b. clinical pathway. c. patient-centered plan of care. d. diagnosis-related group (DRG).

ANS: B A clinical pathway is a plan that specifies the timing and sequencing of major patient care activities and interventions by the interdisciplinary team for a particular diagnosis, procedure, or health condition. pp. 374-375

6. A patient is admitted with coronary artery disease and is scheduled for coronary artery bypass grafting (CABG). According to the clinical pathway the patient should be extubated and discharged from critical care the day after surgery. During surgery the patient's oxygen saturation decreased drastically as a result of chronic tobacco abuse. Subsequently, the patient remained on the ventilator an additional 2 days postoperatively. According to the clinical practice guideline for CABG, this situation represents a: a. patient outcome. b. variance. c. goal. d. standard.

ANS: B A variance is a deviation from the planned path. p. 375

7. An RN delegates to the unlicensed assistive personnel (UAP) the task of performingblood pressure checks for a group of patients on a nursing unit. The UAP accepts the task and is responsible for: A. delegating the task to another UAP if he or she does not have the time or skill to complete the task. B. keeping the RN informed of any abnormal blood pressure readings. C. calling the physician when the patient's vital signs are not within established parameters. D. informing the dietary department to initiate a low-sodium diet for patients who are hypertensive.

ANS: B After accepting the assignment, the UAP is responsible for completing the task and reporting any patient concerns to the RN. p. 354

*29. The most basic factor contributing to the effective supervision of unlicensed assistive personnel (UAP) is the nurse's: A. ability to communicate effectively with others. B. confidence in his or her ability to delegate appropriately. C. experience managing other members of the health care team. D. ability to teach UAPs the necessary skills for their assigned tasks.

ANS: B Because RNs are becoming increasingly responsible for delegation and supervision in today's health care system, it is imperative that they have confidence in their delegation skills and understand the legal responsibility that they assume when delegating to and supervising licensed personnel and UAP. RNs must know what aspects of nursing and health care can be delegated. Although the other options support their supervision of UAP, the basic component is appropriate delegation. p. 353-354

19. A nurse plans care knowing when specific recovery milestones are expected. The nurse is providing care via: a. patient classification systems. b. clinical pathways. c. functional nursing. d. case management.

ANS: B Clinical pathways plans patient care activities and interprofessional interventions and desired patient outcomes within a specified time period for a particular diagnosis or health condition. p. 366

4. A hospital converts to a system of care delivery in which RNs, LPNs, and unlicensed assistive personnel (UAP) are responsible for implementing a specific task, such as medication administration or personal hygiene, for the entire nursing unit. This type of delivery system is: a. total patient care. b. functional nursing. c. team nursing. d. primary nursing.

ANS: B In functional nursing members of the team are assigned specific tasks such as assessment or medication administration. pp. 370-371

*29. The implementation of a "Medication Nurse" is an example of what nursing care delivery model? a. Total patient care b. Functional c. Primary d. Team

ANS: B In the functional nursing method of patient care delivery, staff members are assigned to complete certain tasks for a group of patients rather than care for specific patients. In total patient care, nurses are responsible for planning, organizing, and performing all care for specific clients. In team nursing, the RN functions as a team leader and coordinates a small group (generally no more than four or five) of ancillary personnel to provide care to a small group of patients. In primary nursing, the RN, or "primary" nurse, assumes 24-hour responsibility for planning, directing, and evaluating the patient's care from admission through discharge. pp. 370-371

9. A nurse manager is mentoring a novice nurse manager in determining staffing needs. The mentor explains, "We must determine the acuity level of the patient by: a. assessing patient satisfaction with nursing care." b. quantifying the amount and intensity of care required." c. examining the skill mix and educational preparation of the staff." d. determining the number of hospital days required by the patients."

ANS: B Patient acuity is measured by determining the amount and intensity of care required. p. 367

18. A patient has decided to stop hemodialysis because his renal failure progresses and he wishes to spend more time with family. Palliative care will continue, and the approach will be discussed with the patient and family as needed and at change of shift. The care delivery model in this situation is termed: a. partnership. b. patient-centered. c. case management. d. total patient care.

ANS: B Patient-centered care models entail the health care team partnering with the patient and family to ensure that patients' wants, needs, and preferences are the priority while allowing the patient and family to participate in decisions and educational needs. p. 373

8. The nurse manager determines that four RNs, five LPN/LVNs, and two unlicensed assistive personnel (UAP) are required per shift to meet the needs of the patient population on the unit, according to acuity and census. The nurse manager is concerned with: a. assignments. b. staffing. c. output. d. productivity.

ANS: B Staffing is the activity of determining that an adequate number and mix of health care team members are available to provide safe, high-quality patient care. p. 367

14. A task force is considering factors that contribute to high-quality safe staffing. Which statement reflects an understanding of the American Nurses Association's (ANA) recommendations? a. Because patient needs remain constant on a daily shift, staffing needs at the beginning of the shift should be sufficient to provide safe, high-quality care. b. Staffing should allow time for the RN to apply the nursing process so decisions result in high-quality, safe patient outcomes. c. Patient acuity levels affect staffing by increasing the need for unlicensed personnel to provide routine basic care rather than increasing RNs in staff mix. d. RN staffing is not cost-effective; thus is it important for staffing models to limit the number of RNs assigned per shift.

ANS: B The ANA recommends that nurses have time to exercise professional judgment. p. 367

5. A nurse is delegating to the newly hired nursing unlicensed assistive personnel (UAP) the task of assisting with oral hygiene, knowing that this assignment "does not require decisions based on the nursing process." The nurse is correctly using which of the five rights of delegation? A. Supervision B. Communication C. Person D. Circumstance

ANS: D Right circumstance involves the delegation of tasks that do not require independent nursing judgments. p. 360

11. An RN delegates to an experienced LPN/LVN the task of administering oral medications to a group of patients. The LPN/LVN accepts the assignment, and the RN knows that the LPN/LVN has had the training and has acquired the skills needed to complete the task. The RN then observes the LPN/LVN recording a patient's medication administration just before entering the patient's room. The priority intervention by the RN is to: A. check the patient's drug packages to ensure that the correct drugs were given. B. stop the LPN/LVN immediately and discuss the possible consequences of his actions in a nonjudgmental manner. C. contact the nurse manager and ask that the LPN/LVN's license is suspended. D. call the pharmacy and ask for replacement medications for the patients.

ANS: B The LPN/LVN has the competency but violated one of the rights of medication administration and is practicing unsafe care. The RN's responsibility requires that he or she intervene and identify concerns with the LPN/LVN. p. 363

15. A nursing unit is comparing team nursing to the partnership model and finds that: a. with the partnership model, an RN does not have to be part of the mix. b. leadership abilities of the RN is a major determinant of effectiveness of care for both models. c. the RN teaches the LPN/LVN or unlicensed assistive personnel (UAP) how to apply the nursing process in team nursing. d. with team nursing the RN cares for the patient while the team members work with the family or significant others.

ANS: B The RN leads regardless of whether partnership model or team nursing is practiced. p. 371 |p. 372

6. A student nurse is concerned about delegation practices and wonders why hospitals employ unlicensed assistive personnel (UAP) and LPN/LVNs. The student nurse refers to the National Council of State Boards of Nursing and learns that the role of these personnel is to: A. supplement the staffing pattern when an RN is not available. B. aid the RN by performing appropriately delegated care tasks. C. replace the RN when the health care facility provides long-term care. D. provide patient teaching, allowing more direct care to be provided by the RN.

ANS: B The UAP and LPN/LVN can increase productivity of the RN by performing those tasks that fall within their scope of practice. p. 353

5. The nurse who is responsible for following the patient from admission through discharge or resolution of illness while working with a broad range of health care providers is called a: a. nurse manager. b. case manager. c. coordinator of patient-centered care delivery. d. team leader in team nursing care delivery.

ANS: B The case manager, in collaboration with an interdisciplinary team, oversees the use of health care services by clients throughout a course of illness. p. 373

12. A patient is admitted for a hysterectomy, and the RN develops and implements the plan of care but also delegates to the LPN/LVN the responsibility of administering oral medications. While off duty, this RN receives a call requesting a change in the plan of care because the patient has developed deep vein thrombosis. The nurse who originally planned the care is practicing which type of nursing care delivery? a. Modular b. Primary c. Team d. Functional

ANS: B The primary nurse assumes 24-hour responsibility for planning, directing, and evaluating the patient's care from admission through discharge but may delegate or provide primary care during the shift when present. p. 372

17. Which of the following situations would be appropriate for the supervisory level of initial direction and/or periodic inspection? A. Experienced RNs work together to provide care for a group of patients newly diagnosed with meningitis. B. The RN assigns the LPN tasks within her scope of practice and checks back during the shift to ensure the tasks are completed correctly. C. A new graduate nurse is assigned care to a male patient with a hematocrit of 11.0 g of hemoglobin per deciliter and is receiving a blood transfusion. The charge nurse checks on the patient status every 15 to 30 minutes and asks the graduate to explain "next steps." D. No supervision is necessary since both are registered nurses.

ANS: B When a working relationship is established and competencies of the delegate established, the delegator may check in during intermittently during the shift. p. 361

*33. What are the primary foci for nurses caring for patients currently being admitted to acute care facilities that make traditional nursing models less effective? (Select all that apply.) a. Rescue b. Stabilization c. Prevention d. Transition e. Rehabilitation

ANS: B, D Because acute care settings now admit only the most seriously ill or injured individuals with a focus on stabilization and transition, the traditional models of nursing care may no longer apply. In the past, nurses provided care based on comprehensive knowledge of the patients' needs, which were learned by caring for the patients over an extended period. Now nurses may have an entirely new group of patients to care for every shift or even more than once during a shift. The remaining options while concerns are not typically the focus of the acute care nurse. p. 376

*26. Which statement made by the RN demonstrates a lack of effective delegation to a LPN? *Select all that apply.* A. "Have you ever cared for a postoperative patient who experienced a total mastectomy?" B. "If you begin that complex dressing change while I'm off the floor, get someone to help you." C. "Don't change the patient's catheter until you are sure the correct one has arrived on the unit." D. "Which mandatory competency testing session are you planning to attend?" E. "Remember the patient is depressed so don't say anything to make her more sad."

ANS: B, E Not being available to provide effective supervision and addressing the issue in such a general manner is not a demonstration of effective delegation. Assigning a patient whose emotional state is unstable is not reflective of effective delegation. Evaluating the staff member's competency to perform a task or care for a patient with a particular set of needs and the requirement of mandatory competency testing is a factor in effective delegation. It is necessary to assure that the staff member has the resources including supplies to accomplish the assigned task.REF: p. 360, Box 20-4

*30. An emergency department nurse will likely provide care according to which care delivery model? a. Team b. Primary c. Functional d. Total care

ANS: C Emergency departments often use functional approaches to care because emphasis is on efficient assessment and immediate treatment. The team model is often used in medical-surgical units while the primary care model can be used in long-term care facilities. Some aspects of home health nursing function under the total care model. p. 375

*28. Which factor has the greatest impact on whether safe delegation of nursing tasks is possible? a. The level of competency displayed by the unlicensed assistive personnel (UAP) b. Whether the client is experiencing an acute or a chronic medical health issue c. Whether the client's physical condition is considered stable at the time d. The degree of trust shared between the nurse and the client

ANS: C Generally the more stable the patient, the more likely delegation is to be safe. However, it is also important to remember many tasks that can be delegated may also carry with them a nursing responsibility. The other options are to be considered but they are not as impactful as is the physical condition of the client at the time tasks are being delegated. p. 355

13. When deciding which staffing option to use on a nursing unit that will open soon, the manager realizes that: a. continuity of care is enhanced and errors are reduced when nurses provide care over longer shifts and consecutive workdays, such as 12-hour shifts on 3 consecutive days per week. b. the use of part-time nurses provides the variability needed to meet diverse patient needs. c. satisfaction of the staff equates to satisfaction of patients. d. nurses provide the same level of care, regardless of the work environment.

ANS: C High nurse satisfaction is generally equated with high patient satisfaction and positive patient outcomes. p. 368

14. A nurse moves from California to Arkansas and due to having 20 years of experience as a registered nurse is immediately placed in charge of the telemetry unit. The staffing consists of LPNs and two unlicensed assistive personnel. The RN is unsure of the scope of practice of the LPNs and reviews the nurse practice act for Arkansas, which lacks clarity on some tasks. The RN should: A. query the state nursing association to determine their stance on the role of LPNs. B. ask the LPNs on the unit to list what tasks they routinely performed. C. contact the state board of nursing to determine legal scope of practice for LPNs. D. refer to California's nurse practice act because the scope of LPNs/LVNs is consistent across the United States.

ANS: C If the nurse practice act lacks clarity, the state board of nursing can provide guidance. pp. 354-355

2. Customer satisfaction is primarily based on: a. access to modern, up-to-date facilities. b. availability of an extensive menu selection. c. personal interactions with employees. d. having to undergo fewer invasive procedures.

ANS: C Interactions between employees and patients/families actually affect clinical outcomes, functional status, and even physiologic measures of health. p. 369

*27. The nurse responsible for unit staffing recognize that client care is most affected positively by: a. A mix of high- and low-acuity client needs. b. Units that implement the job-sharing model. c. Nurses who report high levels of job satisfaction. d. Nursing staff that hold advanced nursing degrees.

ANS: C Nurses who are satisfied with their work generally provide higher-quality, more cost-effective care. Staffing systems should consider the quality of work life for the nursing staff as important as the quality of patient outcomes. Although the other options are factors to be considered, they are all influenced by whether the staff is satisfied with their work environment. pp. 368

10. An RN is counseled by the nurse manager regarding inappropriate delegation when the: A. RN instructs the nursing assistant to greet ambulatory surgery patients and show them to their rooms. B. nursing assistant informs the RN that she has not been trained to collect a sputum specimen and the RN states, "I will show you this time and you can show me the next time." C. RN assigns the float LPN/LVN the task of completing a plan of care for a stable patient who was admitted for routine replacement of a feeding tube. D. LPN/LVN who has demonstrated competence is asked to perform a dressing change for a patient before she is discharged home.

ANS: C Only an RN can initiate and complete a new plan of care; this does not fall within the scope of practice of the LPN/LVN. The RN has violated one of the five rights of delegation. p. 357

*28. When considering clinical pathways, the nurse recognizes that an outcome is: a. The result of a collaborative intervention of the healthcare team. b. An event that can prevent or help a patient reach wellness. c. The end result of interventions provided by the health care team. d. An unexpected event that determines a need to change the plan of care

ANS: C Patient outcomes are the end result of interventions by the health care team. Interprofessional intervention is the collaborative effort by all disciplines. Variance is any event that may alter the patient's progress through the clinical pathway. Triggers alert the caregiver that an unexpected event has occurred and a change in the plan of care may be indicated. p. 374-375

15. An RN makes the following assignments at the beginning of the shift. Which assignment would be considered high-risk delegation? A. A novice RN is assigned a patient with diabetes mellitus requiring mixing of regular and NPH insulin. B. An LPN is assigned an older adult with pneumonia and who requires dressing changes on a foot wound. C. An unlicensed assistive person is assigned the task of assisting a patient with late stages of Huntington's disease to ambulate a short distance in the hallway. D. A float RN from the oncology unit is assigned a patient with a white blood cell count of 4000 mm3

ANS: C Risk of falling is great in later stages of Huntington's disease due to chorea movements. pp. 359-360

11. A hospital is concerned with nurse retention and realizes that job satisfaction is a major influence. To enhance employee satisfaction related to staffing, the management team: a. negotiates for additional agency nurses. b. hires more part-time employees. c. includes participatory management into staffing decisions. d. uses "float" nurses to cover vacancies.

ANS: C Staffing methods that include staff participation and enhance staff autonomy have been demonstrated to play a major part in ensuring employee satisfaction. p. 368

*33. After first having a strong understanding of the standards of practice that govern delegation, the registered nurse must know the: a. patient care tasks that are being considered for delegation. b. expected outcomes of the care to be delegated. c. the condition and needs of the patient whose care is being delegated. d. skill and knowledge level of the staff member who is being delegated to

ANS: C The RN must then know the client whose care is being delegated. The client's condition and stability must be determined before tasks and outcomes can be determined or consideration be given to the skill level of the staff being delegated to. p. 358

20. Care delivery using the team-based approach is used on a telemetry nursing unit. The team consists of one registered nurse (RN), two licensed practical nurses (LPNs), and one unlicensed assistive personnel (UAP). Staff have been charged to improve quality of care while ensuring cost containment. Which assignments would meet both criteria? A. The RN administers all medications to all patients. B. The LPN performs sterile dressings and IV tubing changes on all central lines. C. The experienced UAP places telemetry electrodes and attaches to cardiac monitor. D. The RN administers an enema to a stable patient who has an order "administer a fleet enema PRN when no bowel movement in 2 days."

ANS: C The UAP, when properly trained, can place patients on telemetry. This meets quality and cost containment goals because the LPN and RN have higher salaries. pp. 353-354

2. An RN recently relocated to another region of the country and immediately assumed the role of charge nurse. When determining the appropriate person to whom to delegate, the RN knows that: A. the role of the LPN/LVN is the same from state to state. B. the LPN/LVN can be taught to perform all the duties of an RN if approved by the employer and if additional on-the-job training is provided. C. he or she must review the state's nurse practice act for LPN/LVNs, because each state defines the role and scope of practice of the LPN/LVN. D. The Joint Commission has certified and established roles for the LPN/LVN.

ANS: C The scope of practice of the LPN/LVN varies significantly from state to state; RNs should know the LPN/LVN nurse practice act in the state in which they practice and should understand the legal scope of practice of the LPN/LVN. p. 354

9. An LPN/LVN has transferred to a nursing unit and arrives for the first day. The RN checks with the LPN/LVN often throughout the shift to provide support and determine if assistance is needed. The RN is providing which level of supervision? A. There is no supervision because at times the LPN/LVN is not with the RN. B. Periodic inspection is being used. Because the LPN/LVN is licensed, the RN is relieved of the need to evaluate care. C. Continual supervision is being provided until the RN determines competency. D. Initial supervision is being provided because this is the LPN/LVN's first day on the unit.

ANS: C This level of supervision is required when the working relationship is new, the task is complex, or the delegatee is inexperienced or has not demonstrated an acceptable level of competence. p. 362

22. A nursing administrator who is considering the feasibility of an all-RN staff reviews the report, Keeping Patients Safe: Transforming the Work Environment of Nurses (2003) and determines that RNs: *select all that apply* A. are more costly and less efficient than LPNs. B. have little or no effect by being proactive but instead are reactive to patient care errors. C. have a positive effect on patient outcomes when managing patient care. D. are effective overseers of patients' overall health condition. E. lack the training to be effective delegators.

ANS: C, D RNs are effective at coordinating care that results in improved patient outcomes. RNs are valuable monitors of a patient's health status—a practice that results in improved patient outcomes. p. 353

18. A registered nurse (RN) is assigned as charge nurse for the first time. She knows to consult the state board of nursing to determine scope of practice for licensed practical nurses (LPN) and unlicensed assistive personnel (UAP). She also realizes there are common policies which exist in most state practice acts that include: A. the RN is held accountable for the decision to delegate, but responsibility rests only with the delegatee. B. the RN may only delegate tasks that are not in the scope of practice of the LPN if the delegatee is certain they are competent to perform the task. C. since the LPN is licensed, they practice professional nursing. D. to determine what tasks can be safely delegated, the RN must first assess the patient.

ANS: D The stability of the patient must be determined prior to delegation. Even routine tasks such as taking vital signs that are often delegated may need to be performed by the RN when the patient's condition is critical. pp. 354-355, Box 20-1

12. Which statement related to delegation is correct? A. The practice of unlicensed assistive personnel (UAP) is defined in the nurse practice act. B. Nursing practice can be delegated only when the LPN/LVN and UAP have received adequate training. C. Supervision is not required when routine tasks are delegated to a competent individual. D. The RN must be knowledgeable about the laws and regulations that govern nursing practice, as well as those that have no clearly defined parameters, such as for UAP.

ANS: D Accountability remains with the RN, and he or she is responsible for knowing what tasks can be delegated and what is defined as nursing practice. p. 354

1. Accrediting agencies such as The Joint Commission address staffing by: A. imposing maximum staffing levels. B. requiring a specific staff mix. C. stipulating nurse-patient ratios. D. looking for evidence that patients receive satisfactory care.

ANS: D Accrediting agencies do not address minimum staffing levels; however, they do look for evidence that patients receive adequate care, and this can occur only with adequate staffing. p. 369

20. An orthopedic unit is considering different types of care delivery models and staff have an opportunity to ask questions about how the models differ. The nurse manager provides an overview and uses the above visual to demonstrate which model of care delivery? a. Team b. Partnership c. Primary d. Functional

ANS: D Functional care delivery models assign tasks to each provider. In the above visual, the LPN is responsible for oral medication administration, the unlicensed assistive personnel provide hygiene, and the RN is assigned to task that require the nursing process. pp. 370-371

17. A nurse makes patient care assignments as follows: RN1 has rooms 200-210; RN2 has rooms 211-221; RN3 has rooms 222-232. The two unlicensed assistive personnel have half the rooms, with one assigned to 200-215 and the second to 216-232. The care delivery model used in this situation is: a. team. b. primary. c. partnership. d. modular.

ANS: D Modular (or geographic) assignments are based on a geographic location in the nursing unit. p. 372

8. Which task is appropriate for the RN to delegate to the unlicensed assistive personnel (UAP) provided the delegatee has had experience and training? A. Evaluate the ability of a patient to swallow ice after a gastroscopy. B. Assist a patient who has a postoperative hip replacement to ambulate with a walker for the first time. C. Change the disposable tracheotomy cannula for a new postoperative tracheotomy patient if secretions are thick and tenacious. D. Obtain a sterile urine sample from a patient with a Foley catheter that is connected to a closed drainage system.

ANS: D Obtaining a sterile urine sample from a patient with a Foley catheter that is connected to a closed drainage system is not an invasive procedure, and risk to the patient is minimal, making the task appropriate for delegation. pp. 357-358, Box 20-3

*31. Which statement made by an unlicensed assistive personnel (UAP) would cause the registered nurse team leader the most concern? a. "The nurse will follow up to make sure the client is well cared for." b. "I wonder who I can ask about how to use this new electronic thermometer." c. "I've only been working on this unit for a month and things are still new to me." d. "It's good to know that the nurse is really the one responsible for the client's care."

ANS: D The UAP is responsible for his or her own actions. A belief that the nurse is fully responsible is a concern and needs immediate attention by the nurse. It is true that the nurse will follow up and evaluate the client's care. The statements concerning being new and asking for help are appropriate. REF: p. 354

*24. When considering staffing needs, the registered nurse (RN) is primarily concerned with: a. The number of available licensed staff. b. The history of staff absenteeism. c. The availability of support staff. d. Client acuity.

ANS: D The primary considerations for staffing a specific nursing unit are the number of patients followed by the level of intensity of care required by those patients (commonly referred to as patient acuity). Knowing only the number of patients that require care is an ineffective way to plan staffing because of the wide range of care requirements needed by individual patients. The other options may in some degree affect the delivery of care but not to the degree that client acuity does. p. 367

13. During orientation, an RN learns that LPN/LVNs in the facility receive additional training to perform some tasks such as hanging continuously infusing intravenous fluids that have no additives. It is important for the RN to understand that: A. the health care facility can override the state practice act by having all LPN/LVNs and unlicensed assistive personnel (UAP) participate in on-site training. B. LPN/LVNs are licensed, and accountability for their own practice rests with each LPN/LVN. C. UAPs cannot be held responsible for their own actions or inactions. D. the nurse practice act and state regulations related to delegation override the organization's policies.

ANS: D The state's nurse practice act is the deciding factor regarding what can legally be delegated. p. 355

A patient has experienced occasional urinary incontinence in the weeks since his prostatectomy. In order to promote continence, the nurse should encourage which of the following? A) Pelvic floor exercises B) Intermittent urinary catheterization C) Reduced physical activity D) Active range of motion exercises

Ans: A Feedback: Pelvic floor muscles can promote the resumption of normal urinary function following prostate surgery. Catheterization is normally unnecessary, and it carries numerous risks of adverse effects. Increasing or decreasing physical activity is unlikely to influence urinary function.

11. A 42 year-old patient tells the nurse that she has found a painless lump in her right breast during her monthly self-examination. She says that she is afraid that she has cancer. Which assessment finding would most strongly suggest that this patients lump is cancerous? A) Eversion of the right nipple and mobile mass B) A nonmobile mass with irregular edges C) A mobile mass that is soft and easily delineated D) Nonpalpable right axillary lymph nodes

Ans: B Feedback: Breast cancer tumors are typically fixed, hard, and poorly delineated with irregular edges. A mobile mass that is soft and easily delineated is most commonly a fluid-filled benign cyst. Axillary lymph nodes may or may not be palpable on initial detection of a cancerous mass. Nipple retraction, not eversion, may be a sign of cancer.

A 35-year-old man is seen in the clinic because he is experiencing recurring episodes of urinary frequency, dysuria, and fever. The nurse should recognize the possibility of what health problem? A) Chronic bacterial prostatitis B) Orchitis C) Benign prostatic hyperplasia D) Urolithiasis

Ans: A Feedback: Prostatitis is an inflammation of the prostate gland that is often associated with lower urinary tract symptoms and symptoms of sexual discomfort and dysfunction. Symptoms are usually mild, consisting of frequency, dysuria, and occasionally urethral discharge. Urinary incontinence and retention occur with benign prostatic hyperplasia or hypertrophy. The patient may experience nocturia, urgency, decrease in volume and force of urinary stream. Urolithiasis is characterized by excruciating pain. Orchitis does not cause urinary symptoms.

A man tells the nurse that his father died of prostate cancer and he is concerned about his own risk of developing the disease, having heard that prostate cancer has a genetic link. What aspect of the pathophysiology of prostate cancer would underlie the nurse's response? A) A number of studies have identified an association of BRCA-2 mutation with an increased risk of prostate cancer. B) HNPCC is a mutation of two genes that causes prostate cancer in men and it is autosomal dominant. C) Studies have shown that the presence of the TP53 gene strongly influences the incidence of prostate cancer. D) Recent research has demonstrated that prostate cancer is the result of lifestyle factors and that genetics are unrelated.

Ans: A Feedback: A number of studies have identified an association of BRCA-2 mutation with an increased risk of prostate cancer. HPNCC is a form of colon cancer. The TP53 gene is associated with breast cancer.

A nurse is providing care for a patient who has recently been admitted to the postsurgical unit from PACU following a transuretheral resection of the prostate. The nurse is aware of the nursing diagnosis of Risk for Imbalanced Fluid Volume. In order to assess for this risk, the nurse should prioritize what action? A) Closely monitoring the input and output of the bladder irrigation system B) Administering parenteral nutrition and fluids as ordered C) Monitoring the patient's level of consciousness and skin turgor D) Scanning the patient's bladder for retention every 2 hours

Ans: A Feedback: Continuous bladder irrigation effectively reduces the risk of clots in the GU tract but also creates a risk for fluid volume excess if it becomes occluded. The nurse must carefully compare input and output, and ensure that these are in balance. Parenteral nutrition is unnecessary after prostate surgery and skin turgor is not an accurate indicator of fluid status. Frequent bladder scanning is not required when a urinary catheter is in situ.

A patient has just returned to the floor following a transurethral resection of the prostate. A triple-lumen indwelling urinary catheter has been inserted for continuous bladder irrigation. What, in addition to balloon inflation, are the functions of the three lumens? A) Continuous inflow and outflow of irrigation solution B) Intermittent inflow and continuous outflow of irrigation solution C) Continuous inflow and intermittent outflow of irrigation solution D) Intermittent flow of irrigation solution and prevention of hemorrhage

Ans: A Feedback: For continuous bladder irrigation, a triple-lumen indwelling urinary catheter is inserted. The three lumens provide for balloon inflation and continuous inflow and outflow of irrigation solution.

A patient who is postoperative day 12 and recovering at home following a laparoscopic prostatectomy has reported that he is experiencing occasional dribbling of urine. How should the nurse best respond to this patient's concern? A) Inform the patient that urinary control is likely to return gradually. B) Arrange for the patient to be assessed by his urologist. C) Facilitate the insertion of an indwelling urinary catheter by the home care nurse. D) Teach the patient to perform intermittent self-catheterization.

Ans: A Feedback: It is important that the patient know that regaining urinary control is a gradual process; he may continue to dribble after being discharged from the hospital, but this should gradually diminish (usually within 1 year). At this point, medical follow-up is likely not necessary. There is no need to perform urinary catheterization.

A 22-year-old male is being discharged home after surgery for testicular cancer. The patient is scheduled to begin chemotherapy in 2 weeks. The patient tells the nurse that he doesn't think he can take weeks or months of chemotherapy, stating that he has researched the adverse effects online. What is the most appropriate nursing action for this patient at this time? A) Provide empathy and encouragement in an effort to foster a positive outlook. B) Tell the patient it is his decision whether to accept or reject chemotherapy. C) Report the patient's statement to members of his support system. D) Refer the patient to social work.

Ans: A Feedback: Patients may be required to endure a long course of therapy and will need encouragement to maintain a positive attitude. It is certainly the patient's ultimate decision to accept or reject chemotherapy, but the nurse should focus on promoting a positive outlook. It would be a violation of confidentiality to report the patient's statement to members of his support system and there is no obvious need for a social work referral.

To decrease glandular cellular activity and prostate size, an 83-year-old patient has been prescribed finasteride (Proscar). When performing patient education with this patient, the nurse should be sure to tell the patient what? A) Report the planned use of dietary supplements to the physician. B) Decrease the intake of fluids to prevent urinary retention. C) Abstain from sexual activity for 2 weeks following the initiation of treatment. D) Anticipate a temporary worsening of urinary retention before symptoms subside.

Ans: A Feedback: Some herbal supplements are contraindicated with Proscar, thus their planned use should be discussed with the physician or pharmacist. The patient should maintain normal fluid intake. There is no need to abstain from sexual activity and a worsening of urinary retention is not anticipated.

A public health nurse is teaching a health class for the male students at the local high school. The nurse is teaching the boys to perform monthly testicular self-examinations. What point would be appropriate to emphasize? A) Testicular cancer is a highly curable type of cancer. B) Testicular cancer is very difficult to diagnose. C) Testicular cancer is the number one cause of cancer deaths in males. D) Testicular cancer is more common in older men.

Ans: A Feedback: Testicular cancer is highly curable, particularly when it's treated in its early stage. Self-examination allows early detection and facilitates the early initiation of treatment. The highest mortality rates from cancer among men are with lung cancer. Testicular cancer is found more commonly in younger men.

A nurse is teaching a 53-year-old man about prostate cancer. What information should the nurse provide to best facilitate the early identification of prostate cancer? A) Have a digital rectal examination and prostate-specific antigen (PSA) test done yearly. B) Have a transrectal ultrasound every 5 years. C) Perform monthly testicular self-examinations, especially after age 60. D) Have a complete blood count (CBC), blood urea nitrogen (BUN) and creatinine assessment performed annually.

Ans: A Feedback: The incidence of prostate cancer increases after age 50. The digital rectal examination, which identifies enlargement or irregularity of the prostate, and the PSA test, a tumor marker for prostate cancer, are effective diagnostic measures that should be done yearly. Testicular self-examinations won't identify changes in the prostate gland due to its location in the body. A transrectal ultrasound and CBC with BUN and creatinine assessment are usually done after diagnosis to identify the extent of disease and potential metastases.

A 29-year-old patient has just been told that he has testicular cancer and needs to have surgery. During a presurgical appointment, the patient admits to feeling devastated that he requires surgery, stating that it will leave him emasculated and a shell of a man. The nurse should identify what nursing diagnosis when planning the patient's subsequent care? A) Disturbed Body Image Related to Effects of Surgery B) Spiritual Distress Related to Effects of Cancer Surgery C) Social Isolation Related to Effects of Surgery D) Risk for Loneliness Related to Change in Self-Concept

Ans: A Feedback: The patient's statements specifically address his perception of his body as it relates to his identity. Consequently, a nursing diagnosis of Disturbed Body Image is likely appropriate. This patient is at risk for social isolation and loneliness, but there's no indication in the scenario that these diagnoses are present. There is no indication of spiritual element to the patient's concerns.

31. A patient has just returned to the postsurgical unit from post-anesthetic recovery after breast surgery for removal of a malignancy. What is the most likely major nursing diagnosis to include in this patients immediate plan of care? A) Acute pain related to tissue manipulation and incision B) Ineffective coping related to surgery C) Risk for trauma related to post-surgical injury D) Chronic sorrow related to change in body image

Ans: A Feedback: Although many patients experience minimal pain, it is still important to assess for this postsurgical complication. Sorrow and ineffective coping are possible, but neither is likely to be evident in the immediate postoperative period. There is minimal risk of trauma.

24. A nurse is teaching a group of women about the potential benefits of breast self-examination (BSE). The nurse should teach the women that effective BSE is dependent on what factor? A) Womens knowledge of how their breasts normally look and feel B) The rapport that exists between the woman and her primary care provider C) Synchronizing womens routines around BSE with the performance of mammograms D) Womens knowledge of the pathophysiology of breast cancer

Ans: A Feedback: Current practice emphasizes the importance of breast self-awareness, which is a womans attentiveness to the normal appearance and feel of her breasts. BSE does not need to be synchronized with the performance of mammograms. Rapport between the patient and the care provider is beneficial, but does not necessarily determine the effectiveness of BSE. The woman does not need to understand the pathophysiology of breast cancer to perform BSE effectively.

30. A patient has just been told she needs to have an incisional biopsy of a right breast mass. During preoperative teaching, how could the nurse best assess this patient for specific educational, physical, or psychosocial needs she might have? A) By encouraging her to verbalize her questions and concerns B) By discussing the possible findings of the biopsy C) By discussing possible treatment options if the diagnosis is cancer D) By reviewing her medical history

Ans: A Feedback: During the preoperative visit, the nurse assesses the patient for any specific educational, physical, or psychosocial needs that she may have. This can be accomplished by encouraging her to verbalize her fears, concerns, and questions. Reviewing her medical history may be beneficial, but it is not the best way to ascertain her needs. Discussing possible findings of the biopsy and possible treatment options is the responsibility of the treating physician.

19. The nurse is reviewing the physicians notes from the patient who has just left the clinic. The nurse learns that the physician suspects a malignant breast tumor. On palpation, the mass most likely had what characteristic? A) Nontenderness B) A size of 5 mm C) Softness and a regular shape D) Mobility

Ans: A Feedback: Generally, the lesions are nontender, fixed rather than mobile, and hard with irregular borders. Small size is not suggestive of malignancy.

35. A patient has had a total mastectomy with immediate reconstruction. The patient asks the nurse when she can take a shower. What should the nurse respond? A) Not until the drain is removed B) On the second postoperative day C) Now, if you wash gently with soap and water D) Seven days after your surgery

Ans: A Feedback: If immediate reconstruction has been performed, showering may be contraindicated until the drain is removed.

8. A patient who came to the clinic after finding a mass in her breast is scheduled for a diagnostic breast biopsy. During the nurses admission assessment, the nurse observes that the patient is distracted and tense. What is it important for the nurse to do? A) Acknowledge the fear the patient is likely experiencing. B) Describe the support groups that exist in the community. C) Assess the patients stress management skills. D) Document a nursing diagnosis of ineffective coping.

Ans: A Feedback: In the breast cancer diagnostic phase it is appropriate to acknowledge the patients feelings of fear, concern, and apprehension. This must precede interventions such as referrals, if appropriate. Assessment of stress management skills made be necessary, but the nurse should begin by acknowledging the patients feelings. Fear is not necessarily indicative of ineffective coping.

39. A patient who has had a lumpectomy calls the clinic to talk to the nurse. The patient tells the nurse that she has developed a tender area on her breast that is red and warm and looks like someone drew a line with a red marker. What would the nurse suspect is the womans problem? A) Mondor disease B) Deep vein thrombosis (DVT) of the breast C) Recurrent malignancy D) An area of fat necrosis

Ans: A Feedback: Superficial thrombophlebitis of the breast (Mondor disease) is an uncommon condition that is usually associated with pregnancy, trauma, or breast surgery. Pain and redness occur as a result of a superficial thrombophlebitis in the vein that drains the outer part of the breast. The mass is usually linear, tender, and erythematous. Fat necrosis is a condition of the breast that is often associated with a history of trauma. The scenario described does not indicate a recurrent malignancy. DVTs of the breast do not occur.

9. A patient has been referred to the breast clinic after her most recent mammogram revealed the presence of a lump. The lump is found to be a small, well-defined nodule in the right breast. The oncology nurse should recognize the likelihood of what treatment? A) Lumpectomy and radiation B) Partial mastectomy and radiation C) Partial mastectomy and chemotherapy D) Total mastectomy and chemotherapy

Ans: A Feedback: Treatment for breast cancer depends on the disease stage and type, the patients age and menopausal status, and the disfiguring effects of the surgery. For this patient, lumpectomy is the most likely option because the nodule is well-defined. The patient usually undergoes radiation therapy afterward. Because a lumpectomy is possible, mastectomy would not be the treatment of choice.

14. A new mother who is breastfeeding calls the clinic to speak to a nurse. The patient is complaining of pain in her left breast and describes her breast as feeling doughy. The nurse tells her to come into the clinic and be checked. The patient is diagnosed with acute mastitis and placed on antibiotics. What comfort measure should the nurse recommend? A) Apply cold compresses as ordered. B) Avoid wearing a bra until the infection clears. C) Avoid washing the breasts. D) Perform gentle massage to stimulate neutrophil migration.

Ans: A Feedback: Treatment of mastitis consists of antibiotics and local application of cold compresses to relieve discomfort. A broad-spectrum antibiotic agent may be prescribed for 7 to 10 days. The patient should wear a snug bra and perform personal hygiene carefully. Massage is not recommended.

27. A patient is to undergo an ultrasound-guided core biopsy. The patient tells the nurse that a friend of hers had a stereotactic core biopsy. She wants to understand the differences between the two procedures. What would be the nurses best response? A) An ultrasound-guided core biopsy is faster, less expensive, and does not use radiation. B) An ultrasound-guided core biopsy is a little more expensive, but it doesnt use radiation and it is faster. C) An ultrasound-guided core biopsy is a little more expensive, and it also uses radiation but it is faster. D) An ultrasound-guided core biopsy takes more time, and it also uses radiation, but it is less expensive.

Ans: A Feedback: Ultrasound-guided core biopsy does not use radiation and is also faster and less expensive than stereotactic core biopsy.

36. A patient has been discharged home after a total mastectomy without reconstruction. The patient lives alone and has a home health referral. When the home care nurse performs the first scheduled visit this patient, what should the nurse assess? Select all that apply. A) Adherence to the exercise plan B) Overall psychological functioning C) Integrity of surgical drains D) Understanding of cancer E) Use of the breast prosthesis

Ans: A, B, C Feedback: Patients who have difficulty managing their postoperative care at home may benefit from a home health care referral. The home care nurse assesses the patients incision and surgical drain(s), adequacy of pain management, adherence to the exercise plan, and overall physical and psychological functioning. It is unnecessary to assess the patients understanding of cancer at this stage of recovery. Prostheses may be considered later in the recovery process.

The nurse is leading a workshop on sexual health for men. The nurse should teach participants that organic causes of erectile dysfunction include what? Select all that apply. A) Diabetes B) Testosterone deficiency C) Anxiety D) Depression E) Parkinsonism

Ans: A, B, E Feedback: Organic causes of ED include cardiovascular disease, endocrine disease (diabetes, pituitary tumors, testosterone deficiency, hyperthyroidism, and hypothyroidism), cirrhosis, chronic renal failure, genitourinary conditions (radical pelvic surgery), hematologic conditions (Hodgkin disease, leukemia), neurologic disorders (neuropathies, parkinsonism, spinal cord injury, multiple sclerosis), trauma to the pelvic or genital area, alcohol, smoking, medications, and drug abuse. Anxiety and depression are considered to be psychogenic causes.

A 76-year-old with a diagnosis of penile cancer has been admitted to the medical floor. Because the incidence of penile cancer is so low, the staff educator has been asked to teach about penile cancer. What risk factors should the educator cite in this presentation? Select all that apply. A) Phimosis B) Priapism C) Herpes simplex infection D) Increasing age E) Lack of circumcision

Ans: A, D, E Feedback: Several risk factors for penile cancer have been identified, including lack of circumcision, poor genital hygiene, phimosis, HPV, smoking, ultraviolet light treatment of psoriasis on the penis, increasing age (two-thirds of cases occur in men older than 65 years of age), lichen sclerosus, and balanitis xerotica obliterans. Priapism and HSV are not known risk factors.

A patient who is scheduled for an open prostatectomy is concerned about the potential effects of the surgery on his sexual function. What aspect of prostate surgery should inform the nurse's response? A) Erectile dysfunction is common after prostatectomy as a result of hormonal changes. B) All prostatectomies carry a risk of nerve damage and consequent erectile dysfunction. C) Erectile dysfunction after prostatectomy is expected, but normally resolves within several months. D) Modern surgical techniques have eliminated the risk of erectile dysfunction following prostatectomy.

Ans: B Feedback: All prostatectomies carry a risk of impotence because of potential damage to the pudendal nerves. If this damage occurs, the effects are permanent. Hormonal changes do not affect sexual functioning after prostatectomy.

A patient presents to the emergency department with paraphimosis. The physician is able to compress the glans and manually reduce the edema. Once the inflammation and edema subside, what is usually indicated? A) Needle aspiration of the corpus cavernosum B) Circumcision C) Abstinence from sexual activity for 6 weeks D) Administration of vardenafil

Ans: B Feedback: Circumcision is usually indicated after the inflammation and edema subside. Needle aspiration of the corpus cavernosum is indicated in priapism; abstinence from sexual activity for 6 weeks is not indicated. Vardenafil is Levitra and would not be used for paraphimosis

A physician explains to the patient that he has an inflammation of the Cowper glands. Where are the Cowper glands located? A) Within the epididymis B) Below the prostate, within the posterior aspect of the urethra C) On the inner epithelium lining the scrotum, lateral to the testes D) Medial to the vas deferens

Ans: B Feedback: Cowper glands lie below the prostate, within the posterior aspect of the urethra. This gland empties its secretions into the urethra during ejaculation, providing lubrication. The Cowper glands do not lie within the epididymis, within the scrotum, or alongside the vas deferens.

A man comes to the clinic complaining that he is having difficulty obtaining an erection. When reviewing the patient's history, what might the nurse note that contributes to erectile dysfunction? A) The patient has been treated for a UTI twice in the past year. B) The patient has a history of hypertension. C) The patient is 66 years old. D) The patient leads a sedentary lifestyle.

Ans: B Feedback: Past history of infection and lack of exercise do not contribute to impotence. With advancing age, sexual function and libido and potency decrease somewhat, but this is not the primary reason for impotence. Vascular problems cause about half the cases of impotence in men older than 50 years; hypertension is a major cause of such problems.

A 55-year-old man presents at the clinic complaining of erectile dysfunction. The patient has a history of diabetes. The physician orders tadalafil (Cialis) to be taken 1 hour before sexual intercourse. The nurse reviews the patient's history prior to instructing the patient on the use of this medication. What disorder will contraindicate the use of tadalafil (Cialis)? A) Cataracts B) Retinopathy C) Hypotension D) Diabetic nephropathy

Ans: B Feedback: Patients with cataracts, hypotension, or nephropathy will be allowed to take tadalafil (Cialis) and sildenafil (Viagra) if needed. However, tadalafil (Cialis) and sildenafil (Viagra) are usually contraindicated with diabetic retinopathy.

A 75-year-old male patient is being treated for phimosis. When planning this patient's care, what health promotion activity is most directly related to the etiology of the patient's health problem? A) Teaching the patient about safer sexual practices B) Teaching the patient about the importance of hygiene C) Teaching the patient about the safe use of PDE-5 inhibitors D) Teaching the patient to perform testicular self-examination

Ans: B Feedback: Poor hygiene often contributes to cases of phimosis. This health problem is unrelated to sexual practices, the use of PDE-5 inhibitors, or testicular self-examination.

A 35-year-old father of three tells the nurse that he wants information on a vasectomy. What would the nurse tell him about ejaculate after a vasectomy? A) There will be no ejaculate after a vasectomy, though the patient's potential for orgasm is unaffected. B) There is no noticeable decrease in the amount of ejaculate even though it contains no sperm. C) There is a marked decrease in the amount of ejaculate after vasectomy, though this does not affect sexual satisfaction. D) There is no change in the quantity of ejaculate after vasectomy, but the viscosity is somewhat increased.

Ans: B Feedback: Seminal fluid is manufactured predominantly in the seminal vesicles and prostate gland, which are unaffected by vasectomy, thus no noticeable decrease in the amount of ejaculate occurs (volume decreases approximately 3%), even though it contains no spermatozoa. The viscosity of ejaculate does not change.

A patient has been prescribed sildenafil. What should the nurse teach the patient about this medication? A) Sexual stimulation is not needed to obtain an erection. B) The drug should be taken 1 hour prior to intercourse. C) Facial flushing or headache should be reported to the physician immediately. D) The drug has the potential to cause permanent visual changes

Ans: B Feedback: The patient must have sexual stimulation to create the erection, and the drug should be taken 1 hour before intercourse. Facial flushing, mild headache, indigestion, and running nose are common side effects of Viagra and do not normally warrant reporting to the physician. Some visual disturbances may occur, but these are transient.

A patient has returned to the floor from the PACU after undergoing a suprapubic prostatectomy. The nurse notes significant urine leakage around the suprapubic tube. What is the nurse's most appropriate action? A) Cleanse the skin surrounding the suprapubic tube. B) Inform the urologist of this finding. C) Remove the suprapubic tube and apply a wet-to-dry dressing. D) Administer antispasmodic drugs as ordered.

Ans: B Feedback: The physician should be informed if there is significant leakage around a suprapubic catheter. Cleansing the skin is appropriate but does not resolve the problem. Removing the suprapubic tube is contraindicated because it is unsafe. Administering drugs will not stop the leakage of urine around the tube.

18. A nurse is examining a patient who has been diagnosed with a fibroadenoma. The nurse should recognize what implication of this patients diagnosis? A) The patient will be scheduled for radiation therapy. B) The patient might be referred for a biopsy. C) The patients breast mass is considered an age-related change. D) The patients diagnosis is likely related to her use of oral contraceptives.

Ans: B Feedback: Fibroadenomas are firm, round, movable, benign tumors. These masses are nontender and are sometimes removed for biopsy and definitive diagnosis. They are not considered to be an age-related change, even though they are benign. Radiation therapy is unnecessary and fibroadenomas do not result from oral contraceptive use.

25. A 60-year-old man presents at the clinic complaining that his breasts are tender and enlarging. The patient is subsequently diagnosed with gynecomastia. The patient should be assessed for the possibility of what causative factor? A) Age-related physiologic changes B) Medication adverse effects C) Poor nutrition D) Fluid overload

Ans: B Feedback: Gynecomastia can also occur in older men and usually presents as a firm, tender mass underneath the areola. In these patients, gynecomastia may be diffuse and related to the use of certain medications. It is unrelated to fluid overload or nutrition and is not considered an age-related change.

28. A patient at high risk for breast cancer is scheduled for an incisional biopsy in the outpatient surgery department. When the nurse is providing preoperative education, the patient asks why an incisional biopsy is being done instead of just removing the mass. What would be the nurses best response? A) An incisional biopsy is performed because its known to be less painful and more accurate than other forms of testing. B) An incisional biopsy is performed to confirm a diagnosis and so that special studies can be done that will help determine the best treatment. C) An incisional biopsy is performed to assess the potential for recovery from a mastectomy. D) An incisional biopsy is performed on patients who are younger than the age of 40 and who are otherwise healthy.

Ans: B Feedback: Incisional biopsy surgically removes a portion of a mass. This is performed to confirm a diagnosis and to conduct special studies that will aid in determining treatment. Incisional biopsies cannot always remove the whole mass, nor is it always beneficial to the patient to do so. The procedure is not chosen because of the potential for pain, the possibility of recovery from mastectomy, or the patients age.

22. A 42-year-old man has come to the clinic for an annual physical. The nurse notes in the patients history that his father was treated for breast cancer. What should the nurse provide to the patient before he leaves the clinic? A) A referral for a mammogram B) Instructions about breast self-examination (BSE) C) A referral to a surgeon D) A referral to a support group

Ans: B Feedback: Instructions about BSE should be provided to men if they have a family history of breast cancer, because they may have an increased risk of male breast cancer. It is not within the scope of the practice of a nurse to refer a patient for a mammogram or to a surgeon; these actions are not necessary or recommended. In the absence of symptoms or a diagnosis, referral to a support group is unnecessary.

1. A 45-year-old woman comes into the health clinic for her annual check-up. She mentions to the nurse that she has noticed dimpling of the right breast that has occurred in a few months. What assessment would be most appropriate for the nurse to make? A) Evaluate the patients milk production. B) Palpate the area for a breast mass. C) Assess the patients knowledge of breast cancer. D) Assure the patient that this likely an age-related change.

Ans: B Feedback: It would be most important for the nurse to palpate the breast to determine the presence of a mass and to refer the patient to her primary care provider. Edema and pitting of the skin may result from a neoplasm blocking lymphatic drainage, giving the skin an orange-peel appearance (peau dorange), a classic sign of advanced breast cancer. Evaluation of milk production is required in lactating women. There is no indication of lactation in the scenario. The patients knowledge of breast cancer is relevant, but is not a time-dependent priority. This finding is not an age-related change.

17. The nurse is performing a comprehensive health history of a patient who is in her 50s. The nurse should identify what risk factor that may increase this patients risk for breast cancer? A) The patient breastfed each of her children. B) The patient gave birth to her first child at age 38. C) The patient experienced perimenopausal symptoms starting at age 46. D) The patient experienced menarche at age 13.

Ans: B Feedback: Late age at first pregnancy is a risk factor for breast cancer. None of the other listed aspects of the patients health history is considered to be a risk factor for breast cancer.

32. A 52-year-old woman has just been told she has breast cancer and is scheduled for a modified mastectomy the following week. The nurse caring for this patient knows that she is anxious and fearful about the upcoming procedure and the newly diagnosed malignancy. How can the nurse most likely alleviate this patients fears? A) Provide written material on the procedure that has been scheduled for the patient. B) Provide the patient with relevant information about expected recovery. C) Give the patient current information on breast cancer survival rates. D) Offer the patient alternative treatment options

Ans: B Feedback: Providing the patient with realistic expectations about the healing process and expected recovery can help alleviate fears. Offering the patient alternative treatment options is not within the nurses normal scope of practice. Addressing survival rates may or may not be beneficial for the patient. Written material is rarely sufficient to meet patients needs.

A clinic nurse is providing preprocedure education for a man who will undergo a vasectomy. Which of the following measures will enhance healing and comfort? Select all that apply. A) Abstaining from sexual intercourse for at least 14 days postprocedure B) Wearing a scrotal support garment C) Using sitz baths D) Applying a heating pad intermittently E) Staying on bed rest for 48 to 72 hours post-procedure

Ans: B, C Feedback: Applying ice bags intermittently to the scrotum for several hours after surgery can reduce swelling and relieve discomfort, and is preferable to the application of heat. The nurse advises the patient to wear snug, cotton underwear or a scrotal support for added comfort and support. Sitz baths can also enhance comfort. Extended bed rest is unnecessary, and sexual activity can usually be resumed in 1 week.

29. A patient is being discharged home from the ambulatory surgery center after an incisional biopsy of a mass in her left breast. What are the criteria for discharging this patient home? Select all that apply. A) Patient must understand when she can begin ambulating B) Patient must have someone to accompany her home C) Patient must understand activity restrictions D) Patient must understand care of the biopsy site E) Patient must understand when she can safely remove her urinary catheter

Ans: B, C, D Feedback: Prior to discharge from the ambulatory surgical center or the office, the patient must be able to tolerate fluids, ambulate, and void. The patient must have somebody to accompany her home and would not be discharged with urinary catheter in place.

An adolescent is identified as having a collection of fluid in the tunica vaginalis of his testes. The nurse knows that this adolescent will receive what medical diagnosis? A) Cryptorchidism B) Orchitis C) Hydrocele D) Prostatism

Ans: C Feedback: A hydrocele refers to a collection of fluid in the tunica vaginalis of the testes. Cryptorchidism is the most common congenital defect in males, characterized by failure of one or both of the testes to descend into the scrotum. Orchitis is an inflammation of the testes (testicular congestion) caused by pyogenic, viral, spirochetal, parasitic, traumatic, chemical, or unknown factors. Prostatism is an obstructive and irritative symptom complex that includes increased frequency and hesitancy in starting urination, a decrease in the volume and force of the urinary stream, acute urinary retention, and recurrent urinary tract infections.

A nurse is providing an educational event to a local men's group about prostate cancer. The nurse should cite an increased risk of prostate cancer in what ethnic group? A) Native Americans B) Caucasian Americans C) African Americans D) Asian Americans

Ans: C Feedback: African American men have a high risk of prostate cancer; furthermore, they are more than twice as likely to die from prostate cancer as men of other racial or ethnic groups.

A nurse is performing an admission assessment on a 40-year-old man who has been admitted for outpatient surgery on his right knee. While taking the patient's family history, he states, My father died of prostate cancer at age 48. The nurse should instruct him on which of the following health promotion activities? A) The patient will need PSA levels drawn starting at age 55. B) The patient should have testing for presence of the CDH1 and STK11 genes. C) The patient should have PSA levels drawn regularly. D) The patient should limit alcohol use due to the risk of malignancy.

Ans: C Feedback: PSA screening is warranted by the patient's family history and should not be delayed until age 55. The CDH1 and STK11 genes do not relate to the risk for prostate cancer. Alcohol consumption by the patient should be limited. However, this is not the most important health promotion intervention.

A nurse practitioner is assessing a 55-year-old male patient who is complaining of perineal discomfort, burning, urgency, and frequency with urination. The patient states that he has pain with ejaculation. The nurse knows that the patient is exhibiting symptoms of what? A) Varicocele B) Epididymitis C) Prostatitis D) Hydrocele

Ans: C Feedback: Perineal discomfort, burning, urgency, frequency with urination, and pain with ejaculation is indicative of prostatitis. A varicocele is an abnormal dilation of the pampiniform venous plexus and the internal spermatic vein in the scrotum (the network of veins from the testis and the epididymis that constitute part of the spermatic cord). Epididymitis is an infection of the epididymis that usually descends from an infected prostate or urinary tract; it also may develop as a complication of gonorrhea. A hydrocele is a collection of fluid, generally in the tunica vaginalis of the testis, although it also may collect within the spermatic cord.

An uncircumcised 78-year-old male has presented at the clinic complaining that he cannot retract his foreskin over his glans. On examination, it is noted that the foreskin is very constricted. The nurse should recognize the presence of what health problem? A) Bowen's disease B) Peyronie's disease C) Phimosis D) Priapism

Ans: C Feedback: Phimosis is the term used to describe a condition in which the foreskin is constricted so that it cannot be retracted over the glans. Bowen's disease is an in situ carcinoma of the penis. Peyronie's disease is an acquired, benign condition that involves the buildup of fibrous plaques in the sheath of the corpus cavernosum. Priapism is an uncontrolled, persistent erection of the penis from either neural or vascular causes, including medications, sickle cell thrombosis, leukemic cell infiltration, spinal cord tumors, and tumor invasion of the penis or its vessels.

A nurse is caring for a 33-year-old male who has come to the clinic for a physical examination. He states that he has not had a routine physical in 5 years. During the examination, the physician finds that digital rectal examination (DRE) reveals stoney hardening in the posterior lobe of the prostate gland that is not mobile. The nurse recognizes that the observation typically indicates what? A) A normal finding B) A sign of early prostate cancer C) Evidence of a more advanced lesion D) Metastatic disease

Ans: C Feedback: Routine repeated DRE (preferably by the same examiner) is important, because early cancer may be detected as a nodule within the gland or as an extensive hardening in the posterior lobe. The more advanced lesion is stony hard and fixed. This finding is not suggestive of metastatic disease.

A nursing student is learning how to perform sexual assessments using the PLISSIT model. According to this model, the student should begin an assessment by doing which of the following? A) Briefly teaching the patient about normal sexual physiology B) Assuring the patient that what he says will be confidential C) Asking the patient if he is willing to discuss sexual functioning D) Ensuring patient privacy

Ans: C Feedback: The PLISSIT (permission, limited information, specific suggestions, intensive therapy) model of sexual assessment and intervention may be used to provide a framework for nursing interventions. By beginning with the patient's permission, the nurse establishes a patient-centered focus.

A nurse is assessing a patient who presented to the ED with priapism. The student nurse is aware that this condition is classified as a urologic emergency because of the potential for what? A) Urinary tract infection B) Chronic pain C) Permanent vascular damage D) Future erectile dysfunction

Ans: C Feedback: The ischemic form of priapism, which is described as nonsexual, persistent erection with little or no cavernous blood flow, must be treated promptly to prevent permanent damage to the penis. Priapism has not been indicated in the development of UTIs, chronic pain, or erectile dysfunction.

3. A woman aged 48 years comes to the clinic because she has discovered a lump in her breast. After diagnostic testing, the woman receives a diagnosis of breast cancer. The woman asks the nurse when her teenage daughters should begin mammography. What is the nurses best advice? A) Age 28 B) Age 35 C) Age 38 D) Age 48

Ans: C Feedback: A general guideline is to begin screening 5 to 10 years earlier than the age at which the youngest family member developed breast cancer, but not before age 25 years. In families with a history of breast cancer, a downward shift in age of diagnosis of about 10 years is seen. Because their mother developed breast cancer at age 48 years, the daughters should begin mammography at age 38 to 43 years.

37. A patient has just been diagnosed with breast cancer and the nurse is performing a patient interview. In assessing this patients ability to cope with this diagnosis, what would be an appropriate question for the nurse to ask this patient? A) What is your level of education? B) Are you feeling alright these days? C) Is there someone you trust to help you make treatment choices? D) Are you concerned about receiving this diagnosis?

Ans: C Feedback: A trusted ally to assist in making treatment choices is beneficial to the patients coping ability. It is condescending and inappropriate to ask if the patient is feeling alright these days or is concerned about the diagnosis. The patients education level is irrelevant.

21. For which of the following population groups would an annual clinical breast examination be recommended? A) Women over age 21 B) Women over age 25 C) Women over age 40 D) All post-pubescent females with a family history of breast cancer

Ans: C Feedback: Annual clinical breast examination is recommended for women aged 40 years and older. Younger women may have examinations less frequently.

33. A nurse is explaining that each breast contains 12 to 20 cone-shaped lobes. The nurse should explain that each lobe consists of what elements? A) Modified tendons and ligaments B) Connective tissue and smooth muscle C) Lobules and ducts D) Endocrine glands and sebaceous glands

Ans: C Feedback: Each breast contains 12 to 20 cone-shaped lobes, which are made up of glandular elements (lobules and ducts) and separated by fat and fibrous tissue that binds the lobes together. These breast lobes do not consist of tendons, ligaments, endocrine glands, or smooth muscle.

10. A 23-year-old woman comes to the free clinic stating I think I have a lump in my breast. Do I have cancer? The nurse instructs the patient that a diagnosis of breast cancer is confirmed by what? A) Supervised breast self-examination B) Mammography C) Fine-needle aspiration D) Chest x-ray

Ans: C Feedback: Fine-needle aspiration and biopsy provide cells for histologic examination to confirm a diagnosis, although falsenegative and falsepositive findings are possibilities. A breast self-examination, if done regularly, is the most reliable method for detecting breast lumps early, but is not diagnostic of cancer. Mammography is used to detect tumors that are too small to palpate. Chest x-rays can be used to pinpoint rib metastasis. Neither test is considered diagnostic of breast cancer, however.

13. The nurse is caring for a patient who has just had a radical mastectomy and axillary node dissection. When providing patient education regarding rehabilitation, what should the nurse recommend? A) Avoid exercise of the arm for next 2 months. B) Keep cuticles clipped neatly. C) Avoid lifting objects heavier than 10 pounds. D) Use a sling until healing is complete.

Ans: C Feedback: Following an axillary dissection, the patient should avoid lifting objects greater than 5 to 10 pounds, cutting the cuticles, and undergoing venipuncture on the affected side. Exercises of the hand and arm are encouraged and the use of a sling is not necessary.

7. During a recent visit to the clinic a woman presents with erythema of the nipple and areola on the right breast. She states this started several weeks ago and she was fearful of what would be found. The nurse should promptly refer the patient to her primary care provider because the patients signs and symptoms are suggestive of what health problem? A) Peau dorange B) Nipple inversion C) Pagets disease D) Acute mastitis

Ans: C Feedback: Pagets disease presents with erythema of the nipple and areola. Peau dorange, which is associated with breast cancer, is caused by interference with lymphatic drainage, but does not cause these specific signs. Nipple inversion is considered normal if long-standing; if it is associated with fibrosis and is a recent development, malignancy is suspected. Acute mastitis is associated with lactation, but it may occur at any age.

4. A woman scheduled for a simple mastectomy in one week is having her preoperative education provided by the clinic nurse. What educational intervention will be of primary importance to prevent hemorrhage in the postoperative period? A) Limit her intake of green leafy vegetables. B) Increase her water intake to 8 glasses per day. C) Stop taking aspirin. D) Have nothing by mouth for 6 hours before surgery.

Ans: C Feedback: The nurse should instruct the patient to stop taking aspirin due to its anticoagulant effect. Limiting green leafy vegetables will decrease vitamin K and marginally increase bleeding. Increasing fluid intake or being NPO before surgery will have no effect on bleeding.

15. When planning discharge teaching with a patient who has undergone a total mastectomy with axillary dissection, the nurse knows to instruct the patient that she should report what sign or symptom to the physician immediately? A) Fatigue B) Temperature greater than 98.5F C) Sudden cessation of output from the drainage device D) Gradual decline in output from the drain

Ans: C Feedback: The patient should report sudden cessation of output from the drainage device, which could indicate an occlusion. Gradual decline in output is expected. A temperature of 100.4F or greater should also be reported to rule out postoperative infection, but a temperature of 98.5F is not problematic. Fatigue is expected during the recovery period.

2. The nurse leading an educational session is describing self-examination of the breast. The nurse tells the womens group to raise their arms and inspect their breasts in a mirror. A member of the womens group asks the nurse why raising her arms is necessary. What is the nurses best response? A) It helps to spread out the fat that makes up your breast. B) It allows you to simultaneously assess for pain. C) It will help to observe for dimpling more closely. D) This is what the American Cancer Society recommends.

Ans: C Feedback: The primary reason for raising the arms is to detect any dimpling. To elicit skin dimpling or retraction that may otherwise go undetected, the examiner instructs the patient to raise both arms overhead. Citing American Cancer Society recommendations does not address the womans question. The purpose of raising the arms is not to elicit pain or to redistribute adipose tissue.

A patient has returned to the floor after undergoing a transurethral resection of the prostate (TURP). The patient has a continuous bladder irrigation system in place. The patient tells you he is experiencing bladder spasms and asks what you can do to relieve his discomfort. What is the most appropriate nursing action to relieve the discomfort of the patient? A) Apply a cold compress to the pubic area. B) Notify the urologist promptly. C) Irrigate the catheter with 30 to 50 mL of normal saline as ordered. D) Administer a smooth-muscle relaxant as ordered.

Ans: D Feedback: Administering a medication that relaxes smooth muscles can help relieve bladder spasms. Neither a cold compress nor catheter irrigation will alleviate bladder spasms. In most cases, this problem can be relieved without the involvement of the urologist, who will normally order medications on a PRN basis.

A patient has just been diagnosed with prostate cancer and is scheduled for brachytherapy next week. The patient and his wife are unsure of having the procedure because their daughter is 3 months pregnant. What is the most appropriate teaching the nurse should provide to this family? A) The patient should not be in contact with the baby after delivery. B) The patient's treatment poses no risk to his daughter or her infant. C) The patient's brachytherapy may be contraindicated for safety reasons. D) The patient should avoid close contact with his daughter for 2 months.

Ans: D Feedback: Brachytherapy involves the implantation of interstitial radioactive seeds under anesthesia. The surgeon uses ultrasound guidance to place about 80 to 100 seeds, and the patient returns home after the procedure. Exposure of others to radiation is minimal, but the patient should avoid close contact with pregnant women and infants for up to 2 months.

A nurse is planning the postoperative care of a patient who is scheduled for radical prostatectomy. What intraoperative position will place the patient at particular risk for the development of deep vein thrombosis postoperatively? A) Fowler's position B) Prone position C) Supine position D) Lithotomy position

Ans: D Feedback: Elastic compression stockings are applied before surgery and are particularly important for prevention of deep vein thrombosis if the patient is placed in a lithotomy position during surgery. During a prostatectomy, the patient is not placed in the supine, prone, or Fowler's position.

A patient has presented at the clinic with symptoms of benign prostatic hyperplasia. What diagnostic findings would suggest that this patient has chronic urinary retention? A) Hypertension B) Peripheral edema C) Tachycardia and other dysrhythmias D) Increased blood urea nitrogen (BUN)

Ans: D Feedback: Hypertension, edema, and tachycardia would not normally be associated with benign prostatic hyperplasia. Azotemia is an accumulation of nitrogenous waste products, and renal failure can occur with chronic urinary retention and large residual volumes.

A patient confides to the nurse that he cannot engage in sexual activity. The patient is 27 years old and has no apparent history of chronic illness that would contribute to erectile dysfunction. What does the nurse know will be ordered for this patient to assess his sexual functioning? A) Sperm count B) Ejaculation capacity tests C) Engorgement tests D) Nocturnal penile tumescence tests

Ans: D Feedback: Nocturnal penile tumescence tests may be conducted in a sleep laboratory to monitor changes in penile circumference during sleep using various methods to determine number, duration, rigidity, and circumference of penile erections; the results help identify whether the erectile dysfunction is caused by physiologic and/or psychological factors. A sperm count would be done if the patient was complaining of infertility. Ejaculation capacity tests and engorgement tests are not applicable for assessment in this circumstance.

A patient has been diagnosed with erectile dysfunction; the cause has been determined to be psychogenic. The patient's interdisciplinary plan of care should prioritize which of the following interventions? A) Penile implant B) PDE-5 inhibitors C) Physical therapy D) Psychotherapy

Ans: D Feedback: Patients with erectile dysfunction from psychogenic causes are referred to a health care provider or therapist who specializes in sexual dysfunction. Because of the absence of an organic cause, medications and penile implants are not first-line treatments. Physical therapy is not normally effective in the treatment of ED.

A 57-year-old male comes to the clinic complaining that when he has an erection his penis curves and becomes painful. The patient's diagnosis is identified as severe Peyronie's disease. The nurse should be aware of what likely treatment modality? A) Physical therapy B) Treatment with PDE-5 inhibitors C) Intracapsular hydrocortisone injections D) Surgery

Ans: D Feedback: Surgical removal of mature plaques is used to treat severe Peyronie's disease. There is no potential benefit to physical therapy and hydrocortisone injections are not normally used. PDE-5 inhibitors would exacerbate the problem.

A public health nurse has been asked to provide a health promotion session for men at a wellness center. What should the nurse inform the participants about testicular cancer? A) It is most common among men over 55. B) It is one of the least curable solid tumors. C) It typically does not metastasize. D) It is highly responsive to treatment.

Ans: D Feedback: Testicular cancer is most common among men 15 to 35 years of age and produces a painless enlargement of the testicle. Testicular cancers metastasize early but are one of the most curable solid tumors, being highly responsive to chemotherapy.

A patient is 24 hours postoperative following prostatectomy and the urologist has ordered continuous bladder irrigation. What color of output should the nurse expect to find in the drainage bag? A) Red wine colored B) Tea colored C) Amber D) Light pink

Ans: D Feedback: The urine drainage following prostatectomy usually begins as a reddish pink, then clears to a light pink 24 hours after surgery.

23. The nurse is teaching breast self-examination (BSE) to a group of women. The nurse should recommend that the women perform BSE at what time? A) At the time of menses B) At any convenient time, regardless of cycles C) Weekly D) Between days 5 and 7 after menses

Ans: D Feedback: BSE is best performed after menses, on day 5 to day 7, counting the first day of menses as day 1. Monthly performance is recommended.

38. A 35-year-old mother of three young children has been diagnosed with stage II breast cancer. After discussing treatment options with her physician, the woman goes home to talk to her husband, later calling the nurse for clarification of some points. The patient tells the nurse that the physician has recommended breast conservation surgery followed by radiation. The patients husband has done some online research and is asking why his wife does not have a modified radical mastectomy to be sure all the cancer is gone. What would be the nurses best response? A) Modified radical mastectomies are very hard on a patient, both physically and emotionally and they really arent necessary anymore. B) According to current guidelines, having a modified radical mastectomy is no longer seen as beneficial. C) Modified radical mastectomies have a poor survival rate because of the risk of cancer recurrence. D) According to current guidelines, breast conservation combined with radiation is as effective as a modified radical mastectomy.

Ans: D Feedback: Breast conservation along with radiation therapy in stage I and stage II breast cancer results in a survival rate equal to that of modified radical mastectomy. Mastectomies are still necessary in many cases, but are not associated with particular risk of recurrence.

26. A woman is considering breast reduction mammoplasty. When weighing the potential risks and benefits of this surgical procedure, the nurse should confirm that the patient is aware of what potential consequence? A) Chronic breast pain B) Unclear mammography results C) Increased risk of breast cancer D) Decreased nipple sensation

Ans: D Feedback: During the preoperative consultation, the patient should be informed of a possibility that sensory changes of the nipple (e.g., numbness) may occur. There is no consequent increase in breast cancer risk and it does not affect future mammography results. Chronic pain is not an expected complication.

40. A woman calls the clinic and tells the nurse she has had bloody drainage from her right nipple. The nurse makes an appointment for this patient, expecting the physician or practitioner to order what diagnostic test on this patient? A) Breast ultrasound B) Radiography C) Positron emission testing (PET) D) Galactography

Ans: D Feedback: Galactography is a diagnostic procedure that involves injection of less than 1 mL of radiopaque material through a cannula inserted into the ductal opening on the areola, which is followed by mammography. It is performed to evaluate an abnormality within the duct when the patient has bloody nipple discharge on expression, spontaneous nipple discharge, or a solitary dilated duct noted on mammography. X-ray, PET, and ultrasound are not typically used for this purpose.

34. A nurse has assessed that a patient is not yet willing to view her mastectomy site. How should the nurse best assist the patient is developing a positive body image? A) Ask the woman to describe the current appearance of her breast. B) Help the patient to understand that many women have gone through the same unpleasant experience. C) Explain to the patient that her body image does not have to depend on her physical appearance. D) Provide the patient with encouragement in an empathic and thoughtful manner.

Ans: D Feedback: Gentle encouragement can help the patient progress toward accepting the change in her appearance. The nurse should not downplay the significance of physical appearance. Explaining that others have had similar experiences may or may not benefit the patient. Asking the patient to describe the appearance of her breast is likely to exacerbate the womans reluctance to do so.

6. A woman is being treated for a tumor of the left breast. If the patient and her physician opt for prophylactic treatment, the nurse should prepare the woman for what intervention? A) More aggressive chemotherapy B) Left mastectomy C) Radiation therapy D) Bilateral mastectomy

Ans: D Feedback: Right mastectomy would be considered a prophylactic measure to reduce the risk of cancer in the patients unaffected breast. None of the other listed interventions would be categorized as being prophylactic rather than curative.

5. The nurse is caring for a 52-year-old woman whose aunt and mother died of breast cancer. The patient states, My doctor and I talked about Tamoxifen to help prevent breast cancer. Do you think it will work? What would be the nurses best response? A) Yes, its known to have a slight protective effect. B) Yes, but studies also show an increased risk of osteoporosis. C) You wont need to worry about getting cancer as long as you take Tamoxifen. D) Tamoxifen is known to be a highly effective protective measure.

Ans: D Feedback: Tamoxifen has been shown to be a highly effective chemopreventive agent. However, it cannot reduce the risk of cancer by 100%. It also acts to prevent osteoporosis.

16. A patient newly diagnosed with breast cancer states that her physician suspects regional lymph node involvement and told her that there are signs of metastatic disease. The nurse learns that the patient has been diagnosed with stage IV breast cancer. What is an implication of this diagnosis? A) The patient is not a surgical candidate. B) The patients breast cancer is considered highly treatable. C) There is a 10% chance that the patients cancer will self-resolve. D) The patient has a 15% chance of 5-year survival.

Ans: D Feedback: The 5-year survival rate is approximately 15% for stage IV breast cancer. Surgery is still a likely treatment, but the disease would not be considered to be highly treatable. Self-resolution of the disease is not a possibility.

12. A patient in her 30s has two young children and has just had a modified radical mastectomy with immediate reconstruction. The patient shares with the nurse that she is somewhat worried about her future, but she appears to be adjusting well to her diagnosis and surgery. What nursing intervention is most appropriate to support this patients coping? A) Encourage the patients spouse or partner to be supportive while she recovers. B) Encourage the patient to proceed with the next phase of treatment. C) Recommend that the patient remain optimistic for the sake of her children. D) Arrange a referral to a community-based support program.

Ans: D Feedback: The patient is not exhibiting clear signs of anxiety or depression. Therefore, the nurse can probably safely approach her about talking with others who have had similar experiences. The nurse may educate the patients spouse or partner to listen for concerns, but the nurse should not tell the patients spouse what to do. The patient must consult with her physician and make her own decisions about further treatment. The patient needs to express her sadness, frustration, and fear. She cannot be expected to be optimistic at all times.

20. A patient has presented for her annual mammogram. The patient voices concerns related to exposure to radiation. What should the nurse teach the patient about a mammogram? A) It does not use radiation. B) Radiation levels are safe as long as mammograms are performed only once per year. C) The negative effects of radiation do not accumulate until late in life. D) Radiation from a mammogram is equivalent to an hour of sunlight.

Ans: D Feedback: The radiation exposure of mammogram is equivalent to about 1 hour of exposure to sunlight. Consequently, the benefits of mammography far outweigh any risks associated with the procedure. Negative consequences are insignificant, and do not accumulate later in life.

A 24-year-old woman arrives to the emergency department (ED) complaining of a severe headache, nosebleed, and overall weakness and dizziness. She states that she delivered a baby 2 weeks prior, and suffered from hypertension during her entire pregnancy. She delivered 3 weeks early due to a hypertensive crisis. Her vital signs: BP 160/110 mm Hg, HR 118 BPM, RR 24 breaths per minute. When you remove her blood pressure cuff, you note seizure activity and the patient slides to the floor in full tonic-clonic seizure activity. A. ESI Level 1: The patient requires emergent intervention B. ESI Level 2: The patient is ill and falls under the acuity scale C. ESI Level 3: The patient requires several resources D. ESI Level 4: The patient requires seizure precautions

B

A male patient complains of fever, dysuria, and cloudy urine. What additional information may indicate that these manifestations may be something other than a urinary tract infection (UTI)? A. E. coli bacteria in his urine B. A very tender prostate gland C. Complaints of chills and rectal pain D. Complaints of urgency and frequency

B. A very tender prostate gland

A 73-year-old male patient admitted for total knee replacement states during the health history interview that he has no problems with urinary elimination except that the "stream is less than it used to be." The nurse should give the patient anticipatory guidance that what condition may be developing? A. A tumor of the prostate B. Benign prostatic hyperplasia C. Bladder atony because of age D. Age-related altered innervation of the bladder

B. Benign prostatic hyperplasia

A 15-year-old girl presents to the emergency department (ED) with a rash on her ankles and a low grade fever. She has had a headache for one day but is texting on her phone while you obtain a history from her mother. On exam you notice the following: Which ESI level is this patient? Select 1 answer. A. ESI Level 1 B. ESI Level 2 C. ESI Level 3 D. ESI Level 4

C

A 2-year-old boy was bitten by his dog and has a significant laceration to his face, including a large flap over the cheek and upper right lip. He needs a laceration repair. Which ESI level is this patient and why? Select 1 answer. A. ESI 2: Due to the time sensitivity of the facial wound B. ESI 3: Due to the time sensitivity of the facial wound C. ESI 3: He needs a laceration repair and procedural sedation D. ESI 4: He needs a laceration repair

C

Which of the following is an ESI Level 2 patient? Select 1 answer. A. A 38-year-old man with schizophrenia presents because he missed his monthly fluphenazine B. A 52-year-old woman presents because she lost her job, and is tearful; she denies suicidal ideation but would like to see a counselor C. A 21-year-old college student is brought in by his mother. His behavior has become increasingly erratic, and he has been having auditory hallucinations; his roommate called his family, who brought him to the emergency department (ED) D. A 34-year-old woman with a history of alcohol abuse and depression presents after drinking alcohol today. She would like detox; she is not suicidal

C

Which of the following patients is most likely to be inaccurately triaged? Select 1 answer. A. A 45-year-old man with complaint of chest pain and normal vital signs B. A 24-year-old woman, 8 weeks pregnant, with sudden onset left lower quadrant pain and a blood pressure of 85/52 mm Hg C. A 32-year-old woman with a borderline personality disorder D. A 20-day-old female infant with a fever E. A 7-year-old with an obvious deformity to his right wrist after a trip and fall

C

To accurately monitor progression of a symptom of decreased urinary stream, the nurse should encourage the patient to have which primary screening measure done on a regular basis? A. Uroflowmetry B. Transrectal ultrasound C. Digital rectal examination (DRE) D. Prostate-specific antigen (PSA) monitoring

C. Digital rectal examination (DRE)

The patient has had cardiovascular disease for some time and has now developed erectile dysfunction. He is frustrated because he cannot take erectogenic medications because he takes nitrates for his cardiac disease. What should the nurse do first to help this patient? A. Give the patient choices for penile implant surgery. B. Recommend counseling for the patient and his partner. C. Obtain a thorough sexual, health, and psychosocial history. D. Assess levels of testosterone, prolactin, LH, and thyroid hormones.

C. Obtain a thorough sexual, health, and psychosocial history.

The patient has a low-grade carcinoma on the left lateral aspect of the prostate gland and has been on "watchful waiting" status for 5 years. Six months ago his last prostate-specific antigen (PSA) level was 5 ng/mL. Which manifestations now indicate that the prostate cancer may be growing and he needs a change in his care (select all that apply)? A. Casts in his urine B. Presence of α-fetoprotein C. Serum PSA level 10 ng/mL D. Onset of erectile dysfunction E. Nodularity of the prostate gland

C. Serum PSA level 10 ng/mL E. Nodularity of the prostate gland

A patient is one day postoperative following a transurethral resection of the prostate (TURP). Which event is not an expected normal finding in the care of this patient? A. The patient requires two tablets of Tylenol #3 during the night. B. The patient complains of fatigue and claims to have minimal appetite. C. The patient has continuous bladder irrigation (CBI) infusing, but output has decreased. D. The patient has expressed anxiety about his planned discharge home the following day.

C. The patient has continuous bladder irrigation (CBI) infusing, but output has decreased

It is midwinter, and your state health department reports that influenza is widespread in your region. Which of the following patients is an ESI Level 2 patient? Select 1 answer. A. A 43-year-old man presents with 5 days of cough, fever, and headache. He has a fever (38.5°C/101.3°F), pulse oximetry of 97%, and respiratory rate of 14 breaths per minute. B. A 43-year-old man presents with 3 days of cough, fever, and headache. He has a fever (39.1°C/ 103.2°F), pulse oximetry of 97%, and respiratory rate of 14 breaths per minute. C. A 43-year-old man presents with 6 days of cough, fever, and headache. He has a fever (38.5°C/101.3°F), pulse oximetry of 92%, and a respiratory rate of 24 breaths per minute. D. A 43-year-old man presents with 5 days of cough, fever, and headache. He has a fever (38.5°C/101.3°F) and is cyanotic appearing and confused. His oxygen saturation is unable to be detected and his respiratory rate is 4-6 breaths per minute.

D

Which task can the nurse delegate to an unlicensed assistive personnel (UAP) in the care of a patient who has recently undergone prostatectomy? A. Assessing the patient's incision B. Irrigating the patient's Foley catheter C. Assessing the patient's pain and selecting analgesia D. Performing cleansing of the meatus and perineal region

D. Performing cleansing of the meatus and perineal region

"I suddenly started bleeding and passing clots the size of oranges," reports a pale 34-year-old who is 10 days post partum. "I never did this with my other two pregnancies. Can I lie down before I pass out?" Vital signs: BP 86/40, HR 132, RR 22, SpO2 98%.

ESI level 1: Requires immediate lifesaving intervention . This patient is presenting with signs and symptoms of a post partum hemorrhage. She tells you she is going to pass out, and her vital signs reflect her fluid volume deficit. The patient needs immediate IV access and aggressive fluid resuscitation.

A 26-year-old female is transported by EMS to the ED because she experienced the sudden onset of a severe headache that began after she moved her bowels. She is 28 weeks pregnant. Her husband tells you that she is healthy, takes only prenatal vitamins, and has no allergies. On arrival in the ED, the patient is moaning and does not respond to voice. Emergency medical technicians (EMTs) tell you that she vomited about 5 minutes ago.

ESI level 1: Requires immediate lifesaving intervention. From the history, it sounds like this patient has suffered some type of head bleed. She is currently unresponsive to voice and could be showing signs of increased intracranial pressure. She may not be able to protect her own airway and may need to be emergently intubated.

A 58-year-old male collapsed while shoveling snow. Bystander CPR was started immediately; he was defibrillated once by the paramedics with the return of a perfusing rhythm. The hypothermic cardiac arrest protocol was initiated prehospital, and he presents with cold normal saline infusing.

ESI level 1: Requires immediate lifesaving intervention. Studies have shown that lowering brain temperature post cardiac arrest decreases ischemic damage. This patient requires immediate lifesaving interventions to airway, breathing, circulation, and neurologic outcome. Even though the patient converted to a stable rhythm, the nurse should anticipate that additional lifesaving interventions might be necessary.

EMS arrives with a 17-year-old restrained driver involved in a high-speed motor vehicle crash. The patient is immobilized on a backboard and is complaining of abdominal pain. He has multiple lacerations on his left arm. Vital signs prior to arrival: BP 102/60; HR 86, RR 28, SpO2 96%.

ESI level 2: High-risk situation. The mechanism of injury is significant, and this patient has the potential for serious injuries. He needs to be evaluated by the trauma team and should be considered high risk. If his BP was 70/palp and his HR was 128, he would be an ESI level 1; requires immediate life-saving intervention.

A 76-year-old male is brought to the ED because of severe abdominal pain. He tells you, "It feels like someone is ripping me apart." The pain began about 30 minutes prior to admission, and he rates the intensity as 20/10. He has HTN, for which he takes a diuretic. No allergies. The patient is sitting in a wheelchair moaning in pain. His skin is cool and diaphoretic. Vital signs: HR 122, BP 88/68, RR 24, SPO2 94%.

ESI level 1: Requires immediate lifesaving intervention. The patient is presenting with signs of shock—hypotensive, tachycardic, with decreased peripheral perfusion. He has a history of HTN and is presenting with signs and symptoms that could be attributed to a dissecting aortic abdominal aneurysm. He needs immediate IV access, aggressive fluid resuscitation, and perhaps blood prior to surgery.

EMS arrives with a 76-year-old male found on the bathroom floor. The family called 911 when they heard a loud crash in the bathroom. The patient was found in his underwear, and the toilet bowl was filled with maroon-colored stool. Vital signs on arrival: BP 70/palp, HR 128, RR 40. His family tells you he has a history of atrial fibrillation and takes a "little blue pill to thin his blood."

ESI level 1: Requires immediate lifesaving intervention. This 76-year-old patient is in hemorrhagic shock from his GI bleed. His blood pressure is 70, his heart rate is 128, and his respiratory rate is 40, all indicating an attempt to compensate for his blood loss. This patient needs immediate IV access and the administration of fluid, blood, and medications.

"The smoke was so bad; I just couldn't breathe." reports a 26-year-old female who entered her burning apartment building to try to rescue her cat. She is hoarse and complaining of a sore throat and a cough. You notice that she is working hard at breathing. History of asthma; uses inhalers when needed. No known drug allergies. Vital signs: T 98°F, RR 40, HR 114, BP 108/74.

ESI level 1: Requires immediate lifesaving intervention. From the history and presentation, this patient appears to have a significant airway injury and will require immediate intubation. Her respiratory rate is 40, and she is in respiratory distress.

A 72-year-old female with obvious chronic obstructive pulmonary disease and increased work of breathing is wheeled into triage. Between breaths, she tells you that she "is having a hard time breathing and has had a fever since yesterday." The SpO2 monitor is alarming and displaying a saturation of 79 percent.

ESI level 1: Requires immediate lifesaving intervention. Immediate aggressive airway management is what this patient requires. Her saturation is very low, and she appears to be tiring. The triage nurse does not need the other vital signs in order to decide that this patient needs immediate care.

"Nurse, I have this pressure in my chest that started about an hour ago. I was shoveling that wet snow, and I may have overdone it," reports an obese 52-year-old male. He tells you his pain is 10 out of 10 and that he is nauseous and short of breath. His skin is cool and clammy. Vital signs: BP 86/50, HR 52 and irregular.

ESI level 1: Requires immediate lifesaving intervention. The history combined with the signs and symptoms indicate that this patient is probably having an MI. The "pressure" started after shoveling wet snow, and now he is nauseous and short of breath, and his skin is cool and clammy. He needs immediate IV access, the administration of medications, and external pacing pads in place.

"This 84-year-old male passed out in the bathroom," reports the local paramedics. "When we arrived he was in a third-degree heart block with a rate in the 20s and a blood pressure in the 60s. We began externally pacing him at a rate of 60 with an MA in the 50s. He is now alert, oriented, and asking to see his wife."

ESI level 1: Requires immediate lifesaving intervention. The patient is in third-degree heart block and requires external pacing to preserve airway, breathing, and circulation.

Paramedics arrive with a 16-year-old unrestrained driver who hit a tree while traveling at approximately 45 miles per hour. The passenger side of the car had significant damage. The driver was moaning but moving all extremities when help arrived. His initial vital signs were BP 74/50, HR 132, RR 36, SPO2 99%, T 98.6°F.

ESI level 1: Requires immediate lifesaving intervention. The patient is presenting with signs of shock—hypotension, tachycardia, and tachypnea. Based on the mechanism of injury and presenting vital signs, this patient requires immediate lifesaving interventions, including aggressive fluid resuscitation.

EMS arrived with an unresponsive 19-year-old male with a single self-inflicted gunshot wound to the head. Prior to intubation, his Glasgow Coma Scale score was 3.

ESI level 1: Requires immediate lifesaving intervention. The patient is unresponsive and will require immediate lifesaving interventions to maintain airway, breathing, circulation, and neuro status; specifically, the patient will require immediate confirmation of endotracheal tube placement.

EMS presents to the ED with an 18-year-old female with a suspected medication overdose. Her college roommates found her lethargic and "not acting right," so they called 911. The patient has a history of depression. On exam, you notice multiple superficial lacerations to both wrists. Her respiratory rate is 10, and her SpO2 on room air is 86 percent.

ESI level 1: Requires immediate lifesaving intervention. The patient's respiratory rate, oxygen saturation, and inability to protect her own airway indicate the need for immediate endotracheal intubation.

The medical helicopter is en route to your facility with a 16-year-old male who was downhill skiing and hit a tree. Bystanders report that he lost control and hit his head. He was intubated at the scene and remains unresponsive.

ESI level 1: Requires immediate lifesaving interventions. Prehospital intubation is one of the criteria for ESI level 1. This patient has sustained a major head injury and will require an immediate trauma team evaluation.

"I just don't feel right," reports a 21-year-old female who presented in the ED complaining of a rapid heart rate. "I can barely catch my breath, and I have this funny pressure feeling in my chest." HR is 178 and regular, RR 32, BP 82/60. Her skin is cool and diaphoretic.

ESI level 1: Requires immediate lifesaving interventions. The patient is hypotensive with a heart rate of 178. She is showing signs of being unstable—shortness of breath and chest pressure. This patient requires immediate lifesaving interventions, which may include medications and cardioversion.

A 6-year-old male tells you that he was running across the playground and fell. He presents with a 3-centimeter laceration over his right knee. Healthy, no medications and no allergies, immunizations are up to date.

ESI level 4: One resource. The laceration will need to be sutured—one resource.

EMS radios in that they are in route with a 17- year-old with a single gunshot wound to the left chest. On scene the patient was alert, oriented and had a BP of 82/palp. Two large-bore IVs were immediately inserted. Two minutes prior to arrival in the ED, the patient's HR was 130 and BP was 78/palp.

ESI level 1: Requires immediate lifesaving interventions. The trauma team needs to be in the trauma room and ready to aggressively manage this 17-year-old with a single gunshot wound to the left chest. He will require airway management, fluid resuscitation and, depending on the injury, a chest tube or rapid transport to the operating room.

"I just turned my back for a minute," cried the mother of a 4-year-old. The child was pulled out of the family pool by a neighbor who immediately administered mouth-to-mouth resuscitation. The child is now breathing spontaneously but continues to be unresponsive. On arrival in the ED, vital signs were: HR 126, RR 28, BP 80/64, SpO2 96% on a non-rebreather.

ESI level 1: Unresponsive. This 4-year-old continues to be unresponsive. The patient will require immediate lifesaving interventions to address airway, breathing, and circulation.

An 82-year-old resident of a local assisted living facility called 911 because of excruciating generalized abdominal pain and vomiting that started a few hours ago. The woman is moaning in pain but is still able to tell you that she had a heart attack 6 years ago. Vital signs: T 98°F, RR 28, HR 102, BP 146/80, SpO2 98%. Pain 10/10.

ESI level 2: High risk and severe pain and distress. Abdominal pain in the elderly can be indicative of a serious medical condition, and a pain score of 10/10 is significant. The triage nurse needs to keep in mind that due to the normal changes of aging, the elderly patient may present very differently than a younger patient and is more likely to present with vague symptoms.

EMS presents with a 54-year-old female with chronic renal failure who did not go to dialysis yesterday because she was feeling too weak. She tells you to look in her medical record for a list of her current medications and past medical history. Her vital signs are all within normal limits.

ESI level 2: High risk. A complaint of weakness can be due to a variety of conditions, such as anemia or infection. A dialysis patient who misses a treatment is at high risk for hyperkalemia or other fluid and electrolyte problems. This is a patient who cannot wait to be seen and should be given your last open bed.

EMS arrives with an 87-year-old male who fell and hit his head. He is awake, alert, and oriented and remembers the fall. He has a past medical history of atrial fibrillation and is on multiple medications, including warfarin. His vital signs are within normal limits.

ESI level 2: High risk. Patients taking warfarin who fall are at high risk of internal bleeding. Although the patients' vital signs are within normal limits and he shows no signs of a head injury, he needs a prompt evaluation and a head CT.

A 44-year-old female is retching continuously into a large basin as her son wheels her into the triage area. Her son tells you that his diabetic mother has been vomiting for the past 5 hours, and now it is "just this yellow stuff." "She hasn't eaten or taken her insulin," he tells you. No known drug allergies. Vital signs: BP 148/70, P 126, RR 24.

ESI level 2: High risk. A 44-year-old diabetic with continuous vomiting is at risk for diabetic ketoacidosis. The patient's vital signs are a concern, as her heart rate and respiratory rate are both elevated. It is not safe for this patient to wait for an extended period of time in the waiting room.

"I called my pediatrician, and she told me to bring him in because of his fever," reports the mother of a 2-week-old. Vital signs: T 101°F, HR 154, RR 42, SpO2 100%. Uncomplicated, vaginal delivery. The baby is acting appropriately.

ESI level 2: High risk. A temperature higher than 100.4°F (38.0°C) in an infant less than 28 days old is considered high risk no matter how good the infant looks. Infants in this age range are at a high risk for bacteremia.

A 16-year-old male wearing a swimsuit walks into the ED. He explains that he dove into a pool, and his face struck the bottom. You notice an abrasion on his forehead and nose as he tells you that he needs to see a doctor because of tingling in both hands.

ESI level 2: High risk. Because of the mechanism of injury and his complaints of tingling in both hands, this patient should be assigned ESI level 2. He has a cervical spine injury until proven otherwise. He is not an ESI level 1 in that he does not require immediate lifesaving interventions to prevent death. At triage, he needs to be appropriately immobilized.

"My sister has metastatic breast cancer, and her doctor suggested that I bring her in today to have more fluid drained off her lungs." The fluid buildup is making it harder for her to breathe. The patient is a cachectic 42-year-old female on multiple medications. Vital signs: T 98.6°F, RR 34, SpO2 95%, HR 92, BP 114/80.

ESI level 2: High risk. Breast cancer can metastasize to the lungs and can cause a pleural effusion. The collection of fluid in the pleural space leads to increasing respiratory distress as evidenced by the increased respiratory rate and work of breathing.

"I think I'm having a stroke," reports an anxious 40-year-old female. "I looked in the mirror this morning, and the corner of my mouth is drooping and I can't close my left eye. You have to help me, please." No past medical history, no meds. Vital signs all within normal limits.

ESI level 2: High risk. Facial droop is one of the classic signs of a stroke. This patient needs to be evaluated by the stroke team and have a head CT within minutes of arrival in the ED. Many nurses want to make all stroke alerts an ESI level 1. This patient does not meet level 1 criteria as she does not require immediate lifesaving interventions. The triage nurse needs to facilitate moving this patient into the treatment area and initiate the stroke alert process.

A 32-year-old female presents to the emergency department complaining of shortness of breath for several hours. No past medical history, +smoker. Vital signs: RR 32, HR 96, BP 126/80, SpO2 93% on room air, T 98.6°F. No allergies, current medications include vitamins and birth control pills.

ESI level 2: High risk. This 32-year-old female with new-onset shortness of breath is on birth control pills. She is a smoker and is exhibiting signs and symptoms of respiratory distress (SpO2 and respiratory rate.) Based on history and signs and symptoms, a pulmonary embolus, as well as other potential causes for her respiratory distress, must be ruled out.

"I had a baby 5 weeks ago, and I am just exhausted. I have seen my doctor twice, and he told me I wasn't anemic. I climb the stairs, and I am so short of breath when I get to the top that I have to sit down, and now my ankles are swollen. What do you think is wrong with me?" asks a 23-year-old obese female.

ESI level 2: High risk. This patient is describing more than just the fatigue or anemia. This patient could be describing the classic symptoms of a low-volume but high-risk situation—peripartum cardiomyopathy, a form of cardiomyopathy that occurs in the last month of pregnancy and up to 5 months postpartum. There is a decrease in the left ventricular ejection fraction which causes congestive heart failure.

Police escort a disheveled 23-year-old handcuffed male into the triage area. The police report that the patient had been standing in the middle of traffic on the local highway screaming about the end of the world. The patient claims that he had been sent from Mars as the savior of the world. He refuses to answer questions or allow you to take vital signs.

ESI level 2: High risk. This patient is experiencing delusions and may have a past medical history of schizophrenia or other mental illness, or he may be under the influence of drugs. Regardless, the major concern is patient and staff safety. He needs to be taken to a safe, secure area and monitored closely

"My wife called 911 because my internal defibrillator gave me a shock this morning when I was eating breakfast. Really scared me! I saw my doctor a few days ago, and he changed some of my medications. Could that be why that happened?" The patient has a significant cardiac history and reports taking multiple medications, including amiodarone. Vital signs: T 98.5°F, RR 20, HR 90, BP 120/80.

ESI level 2: High risk. This patient is not someone who should sit in your waiting room. He does not meet the criteria for ESI level 1, but he meets the criteria for ESI level 2. The patient's internal defibrillator fired for some reason and needs to be evaluated.

"This is so embarrassing," reports a 42-year-old male. "We were having incredible sex, and I heard a crack. Next thing you know, my penis was flaccid, and I noticed some bruising." The pain is "unbelievable," 20/10. No meds, No known drug allergies.

ESI level 2: High risk. This patient may be describing a penile fracture, a medical emergency. It is most often caused by blunt trauma to an erect penis. This patient needs to be evaluated promptly.

"I was smoking a cigarette and had this coughing fit, and now I feel short of breath," reports a tall, thin 19-year-old man. No past medical history, No meds or allergies, Vital signs: T 98°F, HR 102, RR 36, BP 128/76, SpO2 92%. Pain 0/10.

ESI level 2: High risk. This young, healthy male has an elevated respiratory rate and a low oxygen saturation. The patient's history and signs and symptoms are suggestive of a spontaneous pneumothorax. He needs to be rapidly evaluated and closely monitored.

"My throat is on fire," reports a 19-year-old female. It started a couple of days ago, and it just keeps getting worse. Now I can barely swallow, and my friends say my voice is different. I looked in the mirror, and I have this big swelling on one side of my throat." No past medical history , no meds, no allergies. Vital signs: T 101.6°F, RR24, HR 92, BP 122/80, SpO2 100% on room air.

ESI level 2: High risk. Voice changes, fever, difficulty swallowing, and swelling on one side of the throat can be signs of a peritonsilar abscess. The patient needs to be monitored closely for increasing airway compromise and respiratory distress.

A 28-year-old male presents to the ED requesting to be checked. He has a severe shellfish allergy and mistakenly ate a dip that contained shrimp. He immediately felt his throat start to close, so he used his EpiPen. He tells you that he feels okay. No wheezes or rash noted. Vital signs: BP 136/84, HR 108, RR 20, SpO2 97%, T 97°F.

ESI level 2: High-risk situation for allergic reaction. The patient has used his EpiPen but still requires additional medications and close monitoring.

"I was seen in the ED last night for my fractured wrist. The bone doctor put this cast on and told me to come back if I had any problems. As you can see, my hand is really swollen and the cast is cutting into my fingers. The pain is just unbearable." Circulation, sensation, and movement are decreased.

ESI level 2: High-risk situation. The recent application of a cast along with swelling of the hand and unbearable pain justifies an ESI level-2 acuity level. He may have compartment syndrome.

A 4-year-old female is transported to the ED following a fall off the jungle gym at a preschool. A fall of 4 feet. A witness reports that the child hit her head and was unconscious for a couple of minutes. On arrival you notice that the child is crying and asking for her mother. Her left arm is splinted. Vital signs: HR 162, RR 38.

ESI level 2: High-risk situation. This 4-year-old had a witnessed fall with loss of consciousness and presents to the ED with a change in level of consciousness. She needs to be rapidly evaluated and closely monitored.

This patient is the restrained driver of an SUV involved in a high-speed, multicar accident. Her only complaint is right thigh pain. She has a laceration on her left hand and an abrasion on her left knee. Vital signs: BP 110/74, HR 72, RR 16, no medications, no allergies, no past medical history.

ESI level 2: High-risk situation. Based on mechanism of injury, this patient will need rapid evaluation by the trauma team.

A 17-year-old handcuffed male walks into the ED accompanied by the police. The parents called 911 because their son was out of control: verbally and physically acting out and threatening to kill the family. He is cooperative at triage and answers your questions appropriately. He has no past medical history or allergies and is currently taking no medications. Vital signs are within normal limits.

ESI level 2: High-risk situation. Homicidal ideation is a clear high-risk situation. This patient needs to be placed in a safe, secure environment, even though he is calm and cooperative at triage.

"Why the hell don't you just leave me alone?" yells a 73-year-old disheveled male who was brought to the ED by EMS. He was found sitting on the curb drinking a bottle of vodka with blood oozing from a 4-centimeter forehead laceration. He is oriented to person, place, and time and has a Glasgow Coma Scale score of 14.

ESI level 2: High-risk situation. The history of events is unclear. How did the 73-year-old gentleman get the laceration on his forehead? Did he fall? Get hit? Because of his age, presentation, and presence of alcohol, he is at risk for a number of complications.

An 88-year-old female is brought to the ED by EMS. This morning, she had an episode of slurred speech and weakness of her left arm that lasted about 45 minutes. She has a history of a previous stroke, and she takes an aspirin every day. She is alert and oriented with clear speech and equal hand grasps.

ESI level 2: High-risk situation. The patient's history indicates that she may have had a transient ischemic attack this morning. The patient is high risk, and it would not be safe for her to sit in the waiting room for an extended period of time.

A young male walks into triage and tells you that he has been shot. As he rolls up the left leg of his shorts, you notice two wounds. He tells you that he heard three shots. He is alert and responding appropriately to questions. Initial Vital signs: T 98.2°F, HR 78, RR 16, BP 118/80.

ESI level 2: High-risk situation. This patient has two obvious wounds, but until he is thoroughly examined in the trauma room, you can't rule out the possibility that he has another gunshot wound. The wounds on his thigh look non-life-threatening, but a bullet could have nicked a blood vessel or other structure; therefore, he meets ESI level-2 criteria. His vital signs are within normal limits, so he does not meet ESI level-1 criteria.

EMS arrives with a 75-year-old male with a self-inflicted 6-centimeter laceration to his neck. Bleeding is currently controlled. With tears in his eyes, the patient tells you that his wife of 56 years died last week. Health, No known drug allergies, baby ASA per day, BP 136/82, HR 74, RR 18, SpO2, 98% RA.

ESI level 2: High-risk. This 75-year-old male tried to kill himself by cutting his throat. Because of the anatomy of the neck, this type of laceration has the potential to cause airway, breathing, and/or circulation problems. At the same time, he is suicidal, and the ED needs to ensure that he does not leave or attempt to harm himself further.

"My mother is just not acting herself," reports the daughter of a 72-year-old female. She is sleeping more than usual and complains that it hurts to pee." Vital signs: T 100.8°F, HR 98, RR 22, BP 122/80. The patient responds to verbal stimuli but is disoriented to time and place.

ESI level 2: New onset confusion, lethargy, or disorientation. The daughter reports that her mother has a change in level of consciousness. The reason for her change in mental status may be a UTI that has advanced to bacteremia. She has an acute change in mental status and is therefore high risk.

A 20-year-old male presents to the ED after being tackled while playing football. He has an obvious dislocation of his left shoulder and complains of 10/10, severe pain. Neurovascular status is intact, and vital signs are within normal limits.

ESI level 2: Severe pain and distress. The triage nurse is unable to manage his pain at triage other than applying a sling and ice. He will require IV opioids to reduce his pain and relocate his shoulder.

The patient states that she is 6 weeks post laparoscopic gastric bypass. Two days ago, she began to have abdominal pain with nausea and vomiting of pureed food. She reports a decrease in her fluid intake and not being able to take her supplements because of vomiting. Vital signs: T 97.8°F, RR 20, HR 90, BP 110/70, SpO2 99%. Pain 4/10.

ESI level 3: Two or more resources . Abdominal pain and vomiting post gastric bypass needs to be evaluated. This patient needs labs, IV, antiemetics, and a CT.

"He was running after his brother, fell, and cut his lip on the corner of the coffee table. There was blood everywhere," recalls the mother of a healthy 19-month-old. "He'll never stay still for the doctor." You notice that the baby has a 2- centimeter lip laceration that extends through the vermilion border. Vital signs are within normal limits for age.

ESI level 3: Two or more resources. A laceration through the vermilion border requires the physician to line up the edges exactly. Misalignment can be noticeable. A healthy 19-month-old will probably not cooperate. In most settings, he will require conscious sedation, which counts as two resources. The toddler's vital signs are within normal limits for his age, so there is no reason to up-triage to ESI level 2.

"Our pediatrician told us to bring the baby to the emergency department to see a surgeon and have some tests. Every time I feed him, he vomits and it just comes flying out," reports the mother of a healthy appearing 3�week-old. "None of my other kids did this." Normal vaginal delivery. Vital signs are within normal limits.

ESI level 3: Two or more resources. A 3-week-old with projectile vomiting is highly suspicious for pyloric stenosis. The infant will need, at minimum, labs to rule out electrolyte abnormalities, an ultrasound, and a surgery consult.

A 58-year-old male presents to the emergency department complaining of left lower-quadrant abdominal pain for 3 days. He denies nausea, vomiting, or diarrhea. No change in appetite. past medical history HTN. Vital signs: T 100°F, RR 18, HR 80, BP 140/72, SpO2 98%. Pain 5/10.

ESI level 3: Two or more resources. Abdominal pain in a 58-year-old male will require two or more resources. At a minimum, he will need labs and an abdominal CT.

A normal healthy 7-year-old walks into the emergency department accompanied by his father, who reports that his son woke up complaining of a stomach ache. "He refused to walk downstairs and is not interested in eating or playing." The child vomits at triage. Vital signs: T 100.4°F, RR 22, HR 88, BP 84/60, SpO2100%. Pain 6/10.

ESI level 3: Two or more resources. At a minimum, this child will need a workup for his abdominal pain, which will include labs and a CT or ultrasound—two resources.

"How long am I going to have to wait before I see a doctor?" asks a 27-year-old female with a migraine. The patient is well known to you and your department. She rates her pain as 20/10 and tells you that she has been like this for 2 days. She vomited twice this morning. past medical history: migraines, no allergies, medications include Fioricet.

ESI level 3: Two or more resources. At a minimum, this patient will require an IV with fluid, IV pain medication, and an antiemetic. Although she rates her pain as 20/10, she should not be assigned to ESI level 2. She has had the pain for 2 days, and the triage nurse can't justify giving the last open bed to this patient. The triage nurse will need to address this patient's concerns about wait time.

"I got my belly button pierced a month ago and now it hurts so bad," reports a 19-year-old healthy college student who is accompanied by her roommate. They are chatting about plans for the evening. The area is red, tender, and swollen, and pus is oozing from around the site. Vital signs: T 100°F, HR 74, RR 18, BP 102/70, SpO2 100%. Pain 8/10.

ESI level 3: Two or more resources. Based on the history, this patient may have a cellulitis from the navel piercing. At a minimum she will require labs and IV antibiotics.

EMS arrives with a 42-year-old male who called 911 because of dizziness and nausea every time he tries to move. The patient states, "I feel okay when I lie perfectly still, but if I start to sit up, turn over, or move my head, the room starts to spin and I have to throw up." No past medical history. Vital signs: T 97.2°F, RR 16, HR 90, BP 130/82, SpO2 99%. Pain 0/10.

ESI level 3: Two or more resources. Based on the history, this patient may have acute labyrinthitis and will require two or more resources—IV fluids and an IV antiemetic.

"I think I have food poisoning," reports an otherwise healthy 33-year-old female. "I have been vomiting all night, and now I have diarrhea." The patient admits to abdominal cramping that she rates as 5/10. She denies fever or chills. Vital signs: T 96.8°F, HR 96, RR 16, BP 116/74.

ESI level 3: Two or more resources. Lab studies, IV fluid, and an IV antiemetic are three of the resources this patient will require. The patient is not high risk or in severe pain or distress.

A 60-year-old man requests to see a doctor because his right foot hurts. On exam the great toe and foot skin is red, warm, swollen, and tender to touch. He denies injury. past medical history includes type 2 diabetes, and psoriasis. Vital signs: T 99.4°F, RR 18, HR 82, BP 146/70, SpO2 99%.

ESI level 3: Two or more resources. This patient has a significant medical history, and based on his presentation, he will require two or more resources, which could include labs and IV antibiotics.

"My right leg is swollen, and my calf hurts," reports a 47-year-old morbidly obese female sitting in a motorized scooter. The patient denies chest pain or shortness of breath, but admits to a history of type 2 diabetes and HTN. Vital signs: T 98°F, RR 24, HR 78, BP 158/82, SpO2 98%. Pain 6/10.

ESI level 3: Two or more resources. This patient is at high risk for a deep vein thrombosis. For diagnostic purposes, she will require two resources: labs and a Doppler ultrasound. If a deep vein thrombosis is confirmed, she will require additional resources—remember, ESI level 3 is two or more resources. If this patient were short of breath or had chest pain, they would meet ESI level-2 criteria.

"I have been on antibiotics for 5 days for mastitis. I am continuing to nurse my baby, but I still have pain and tenderness in my right breast. Now there is this new reddened area," a 34-year-old new mother tells you. The patient reports having a fever, chills, and just feeling run down. T 102.2°F, RR 20, HR 990, BP 122/80, SpO2 98%. Pain 6/10.

ESI level 3: Two or more resources. This patient probably has been on antibiotics for 5 days for mastitis and now presents to the ED due to fever, chills, and feeling rundown. She will require labs, IV antibiotics, a lactation consult if available, and perhaps admission.

"My migraine started early this morning, and I can't get it under control. I just keep vomiting. Can I lie down somewhere?" asks a 37-year-old female. Past medical history migraines, no allergies. Pain 6/10, T 98°F, RR 20, HR 102, BP 118/62, SpO2 98%.

ESI level 3: Two or more resources. A patient with a known history of migraines with vomiting will require pain medication, an antiemetic, and fluid replacement. The pain is not severe, 6/10. This patient is not high risk.

EMS arrives with a 32-year-old female who fell off a stepladder while cleaning her first-floor gutters. She has an obvious open fracture of her right lower leg. She has +2 pedal pulse. Her toes are warm, and she is able to wiggle them. Denies past medical history medications, or allergies. Vital signs are within normal limits for her age.

ESI level 3: Two or more resources. An obvious open fracture will necessitate this patient going to the operating room. At a minimum, she will need the following resources: x-ray, lab, IV antibiotics, and IV pain medication.

"My right breast is so sore, my nipples are cracked, and now I have a fever. Do you think I will have to stop nursing my baby?" asks a tearful 34-year-old female. She is 3 months post partum and has recently returned to work parttime. Vital signs: T 102.8°F, HR 90, RR 18, BP 108/60, pain 5/10. No past medical history, taking multivitamins, and is allergic to penicillin.

ESI level 3: Two or more resources. At a minimum, she will require labs and IV antibiotics.

"I have this aching pain in my left leg," reports an obese 52-year-old female. "The whole ride home, it just ached and ached." The patient tells you that she has been sitting in a car for the last 2 days. "We drove my daughter to college, and I thought it was the heat getting to me." She denies any other complaints. Vital signs: BP 148/90, HR 86, RR 16, T 98°F.

ESI level 3: Two or more resources. At a minimum, she will require labs and noninvasive vascular studies of her lower leg. She should be placed in a wheelchair with her leg elevated and instructed not to walk until the doctor has seen her.

A 41-year-old male involved in a bicycle accident walks into the emergency department with his right arm in a sling. He tells you that he fell off his bike and landed on his right arm. His is complaining of pain in the wrist area and has a 2-centimeter laceration on his left elbow. "My helmet saved me," he tells you.

ESI level 3: Two or more resources. At a minimum, this patient will require an x-ray of his right arm and suturing of his left elbow laceration.

A 27-year-old female wants to be checked by a doctor. She has been experiencing low abdominal pain (6/10) for about 4 days. This morning, she began spotting. She denies nausea, vomiting, diarrhea, or urinary symptoms. Her last menstrual period was 7 weeks ago. past medical history: previous ectopic pregnancy. Vital signs: T.98°F, HR 66, RR 14, BP 106/68.

ESI level 3: Two or more resources. Based on her history, this patient will require two or more resources—lab and an ultrasound. She may in fact be pregnant. Ectopic pregnancy is on the differential diagnosis list, but this patient is currently hemodynamically stable, and her pain is generalized across her lower abdomen.

"It is like I have my period. I went to the bathroom, and I am bleeding. This is my first pregnancy, and I am scared. Do you think everything is OK?" asks a 26-year-old healthy female. Vital signs: BP 110/80, HR 72, RR 18, SpO2 99%, T 98.6°F. She describes the pain as crampy, but rates it as "1" out of 10.

ESI level 3: Two or more resources. Based on her history, this patient will require two or more resources—labs, an ultrasound. On the differential diagnosis list is a spontaneous abortion. Currently, she is hemodynamically stable and has minimal cramping or pain.

A-25-year-old female presented to the emergency department because of moderate lower abdominal pain with a fever and chills. Two days ago, the patient had a therapeutic abortion at a local clinic. The patient reports minimal vaginal bleeding, Vital signs: T 100.8°F, RR 20, HR 92, BP 118/80, SpO2 99%. Pain 5/10.

ESI level 3: Two or more resources. Based on the history, this patient will require at a minimum labs and IV antibiotics. In addition she may need a gyn consult and IV pain medication.

A healthy middle-aged man presents to the emergency department with his left hand wrapped in a bloody cloth. "I was using my table saw, and my hand slipped. I think I lost of couple of fingertips." No past medical history, no med or allergies. Vital signs are within normal limits. Pain 6/10.

ESI level 3: Two or more resources. Based on the patient's presentation, he will require a minimum IV pain medication and laceration repairs. In addition he may need an x-ray and IV antibiotics.

A 68-year-old female presents to the ED with her right arm in a sling. She was walking out to the mailbox and slipped on the ice. "I put my arm out to break my fall. I was lucky I didn't hit my head." Right arm with good circulation, sensation, and movement, obvious deformity noted. past medical history: arthritis, medications, ibuprofen, No known drug allergies. Vital signs within normal limits. She rates her pain as 6/10.

ESI level 3: Two or more resources. It looks like this patient has a displaced fracture and will need to have a closed reduction prior to casting or splinting. At a minimum, she needs x-rays and an orthopedic consult. Her vital signs are stable, so there is no need to uptriage her to an ESI level 2. Her pain is currently 6/10. If she rated her pain as 9/10 and she is tearful, would you up-triage her to an lESI level 2? Probably not, given the many nursing interventions you could initiate to decrease her pain, such as ice, elevation, and appropriate immobilization.

"I'm 7 weeks pregnant, and every time I try to eat something, I throw up," reports a 27-year-old female. "My doctor sent me to the emergency department because he thinks I am getting dehydrated. T 97°F, RR 18, HR 104, BP 104/68, SpO2 99%. Pain 0/10. Lips are dry and cracked.

ESI level 3: Two or more resources. Lab studies, IV fluid, and an IV antiemetic are three of the resources this patient will require. She is showing signs of dehydration.

"My doctor told me to come to the ED. I had a gastric bypass 3 weeks ago and have been doing fine, but today I started vomiting and having this belly pain." The patient, an obese 33-yearold female, rates her pain as 6/10. Vital signs: BP 126/70, HR 76, RR 14, T 98°F.

ESI level 3: Two or more resources. She will need two or more resources—laboratory tests, IV fluid, medication for her nausea, and probably a CT of her abdomen. This patient will be in your emergency department an extended period of time being evaluated. If her pain was 10/10 and she was tachycardic, the patient would meet the ESI level-2 criteria.

EMS arrives with a 45-year-old woman with asthma who has had a cold for week. She started wheezing a few days ago and then developed a cough and a fever of 103. Vital signs: T 101.6°F, HR 92, RR 24, BP 148/86, SpO2 97%.

ESI level 3: Two or more resources. This history sounds more like pneumonia. Because the patient is not in acute respiratory distress, he or she doesn't meet ESI level-2 criteria. This patient will require labs, a chest x-ray, and perhaps IV antibiotics.

"My doctor told me to come to the ED. He thinks my hand is infected," a 76-year-old female with arthritis, chronic renal failure, and diabetes tells you. She has an open area on the palm of her hand that is red, tender, and swollen. She hands you a list of her medications and reports that she has no allergies. She is afebrile. Vital signs: HR 72, RR 16, BP 102/60.

ESI level 3: Two or more resources. This patient has a complex medical history and presented with an infected hand. At a minimum she will need labs, an IV, and IV antibiotics to address her presenting complaint. Her vital signs are normal, so there is no reason to up-triage her to ESI level 2.

A 63-year-old cachectic male is brought in from the local nursing home because his feeding tube fell out again. The patient is usually unresponsive. He has been in the nursing home since he suffered a massive stroke about 4 years ago.

ESI level 4: One resource. This patient will be sent back to the nursing home after the feeding tube is reinserted. There is no acute change in his medical condition that warrants any further evaluation. He is unresponsive, but that is the patient's baseline mental status so he is not an ESI level 1.

"I have had a cold for a few days, and today I started wheezing. When this happens, I just need one of those breathing treatments," reports a 39-year-old female with a history of asthma. T 98°F, RR 22, HR 88, BP 130/80, SpO2 99%, No meds, no allergies.

ESI level 4: One resource. This patient will need a hand-held nebulizer treatment for her wheezing. No labs or x-ray should be necessary because the patient does not have a fever.

"I have a fever and a sore throat. I have finals this week, and I am scared this is strep," reports a 19-year-old college student. She is sitting at triage drinking bottled water. No past medical history, medications: birth control pills, no allergies to medications. Vital signs: T 100.6°F, HR 88, RR 18, BP 112/76.

ESI level 4: One resource. In most EDs, this patient will have a rapid strep screen sent to the lab; one resource. She is able to drink fluids and will be able to swallow pills if indicated.

A 52-year-old female requests to see a doctor for a possible urinary tract infection (UTI). She is complaining of dysuria and frequency. She denies abdominal pain or vaginal discharge. No allergies, takes vitamins, and has no significant past medical history. Vital signs: T 97.4°F, HR 78, RR 14, BP 142/70.

ESI level 4: One resource. She will need one resource—lab, which will include a urinalysis and urine culture. She most likely has a UTI that will be treated with oral medications.

"I was at a family reunion, and we were playing baseball. One of my nephews hit the ball so hard, and I tried to catch it, missed, and it hit me right in the eye. My vision is fine. It just hurts," reports a 34-year-old healthy female. Vital signs are within normal limits. There are no obvious signs of trauma to the globe, only redness and swelling in the periorbital area. The patient denies loss of consciousness.

ESI level 4: One resource. The history is suggestive of an orbital fracture. The patient will require one resource—an x-ray. She will need a visual acuity check and eye evaluation, but these are not ESI resources.

A 26-year-old female walks into the triage room and tells you she needs to go into detox again. She has been clean for 18 months but started using heroin again 2 weeks ago when her boyfriend broke up with her. She had called several detox centers but was having no luck finding a bed. She denies suicidal or homicidal ideation. She is calm and cooperative.

ESI level 4: One resource. This patient is seeking help finding a detoxification program that will help her. She is not a danger to herself or others. The social worker or psychiatric counselor should be consulted to assist her. Once a placement has been found, she can be discharged from the emergency department and can get herself to the outpatient program. If your social worker or psychiatric counselor requires a urine toxicology or other lab work, the patient will require two or more resources and then meet ESI level-3 criteria.

"I slipped on the ice, and I hurt my wrist," reports a 58-year-old female with a history of migraines. There is no obvious deformity. Vital signs are within normal limits, and she rates her pain as 5/10.

ESI level 4: One resource. This patient needs an x-ray to rule out a fracture. A splint is not a resource.

"I cut my finger trying to slice a bagel," reports a 28-year-old healthy male. A 2-centimeter laceration is noted on the left first finger. Bleeding is controlled. Vital signs are within normal limits. His last tetanus immunization was 10 years ago.

ESI level 4: One resource. This patient will require a laceration repair. A tetanus booster is not a resource.

"I should have paid more attention to what I was doing," states a 37-year-old carpenter who presents to the ED with a 3-centimeter laceration to his right thumb. The thumb is wrapped in a clean rag. "I know I need a tetanus shot," he tells you. BP 142/76, RR 16, T 98.6°F.

ESI level 4: One resource. This patient will require a laceration repair. A tetanus booster is not a resource.

"I was using my chainsaw without safety goggles, and I think I got some sawdust in my left eye. It hurts and it just won't stop tearing," reports a healthy 36-year-old male. Vital signs are within normal limits.

ESI level 4: One resource. This patient will require eye irrigation. Eye drops are not a resource. A slit lamp exam is part of the physical exam of this patient.

"It hurts so much when I urinate," reports an otherwise healthy 25-year-old. She denies fever, chills, abdominal pain, or vaginal discharge. Vital signs: T 98.2°F, HR 66, RR 14, BP 114/60.

ESI level 4: One resource. This patient will require one resource—lab. A urinalysis and urine culture will be sent, and depending on your institution, a urine pregnancy test. One or all of these tests count as one resource.

"I have this infection in my cuticle," reports a healthy 26-year-old female. "It started hurting 2 days ago, and today I noticed the pus." The patient has a small paronychia on her right second finger. No known drug allergies. T 98.8°F, RR 14, HR 62, BP 108/70.

ESI level 4: One resource. This young woman needs an incision and drainage of her paronychia. She will require no other resources.

"My dentist can't see me until Monday, and my tooth is killing me. Can't you give me something for the pain?" a healthy 38-year-old male asks the triage nurse. He tells you the pain started yesterday, and he rates his pain as 10/10. No obvious facial swelling is noted. Allergic to penicillin. Vital signs: T 99.8°F, HR 78, RR 16, BP 128/74.

ESI level 5. No resources. No resources should be necessary. He will require a physical exam, but without signs of an abscess or cellulitis, this patient will be referred to a dentist. In the ED, he may be given oral medications and prescriptions for antibiotics and/or pain medication. He is not an ESI level 2, even though he rates his pain as 10/10. Based on the triage assessment, he would not be given the last open bed.

You are trying to triage an 18-month-old whose mother brought him in for vomiting. The toddler is very active and trying to get off his mother's lap. To distract him, the mother hands him a bottle of juice, which he immediately begins sucking on. The child looks well hydrated and is afebrile.

ESI level 5: No resources. A physical exam and providing the mother with reassurance and education is what this 18-month-old will require. His activity level is appropriate, and he is taking fluids by mouth.

"My son woke me up about 3 hours ago complaining of a right earache. I gave him some acetaminophen but it didn't help," the 4-year-old's mother tells you. No fever, other vital signs within normal limits for age.

ESI level 5: No resources. Following a physical exam, this 4-year-old will be sent home with appropriate discharge instructions and perhaps a prescription.

"I have this rash in my groin area," reports a 20-year-old healthy male. "I think it's jock rot, but I can't get rid of it." Using over the counter spray. No known drug allergies. Vital Signs: T 98°F, HR 58, RR 16, BP 112/70.

ESI level 5: No resources. Following a physical exam, this patient will be sent home with prescriptions and appropriate discharge instructions.

"I was taking my contacts out last night, and I think I scratched my cornea," reports a 27-year-old female. "I'm wearing these sunglasses because the light really bothers my eyes." Her right eye is red and tearing. She rates her pain as 6/10. Vital signs are within normal limits.

ESI level 5: No resources. This patient will need an eye exam and will be discharged to home with prescriptions and an appointment to follow up with an ophthalmologist.

"I think I need a tetanus shot," a 29-year-old female tells you. "I stepped on a rusty nail this morning, and I know I haven't had one for years." No past medical history, No known drug allergies, no medications.

ESI level 5: No resources. A tetanus immunization does not count as a resource. The patient will be seen by a physician or midlevel provider and receive a tetanus immunization and discharge instructions. This patient will require no resources.

"My son needs a physical for camp," an anxious mother tells you. "I called the clinic, but they can't see him for 2 weeks and camp starts on Monday." Her son, a healthy 9-yearold, will be attending a summer day camp.

ESI level 5: No resources. Because the mother could not get an appointment with a primary care physician, she brought her son to the emergency department for a routine physical exam. He will be examined and discharged.

Concerned parents arrive in the ED with their 4-day-old baby girl who is sleeping peacefully in the mother's arms. "I went to change her diaper," reports the father, "and I noticed a little blood on it. Is something wrong with our daughter?" The mother tells you that the baby is nursing well and weighed 7 lbs., 2 oz. at birth.

ESI level 5: No resources. The parents of this 4-day-old need to be reassured that a spot of blood on their baby girl's diaper is not uncommon. The baby is nursing and looks healthy.

"I ran out of my blood pressure medicine, and my doctor is on vacation. Can someone here write me a prescription?" requests a 56-year-old male with a history of HTN. Vital signs: BP 128/84, HR 76, RR 16, T 97°F.

ESI level 5: No resources. The patient needs a prescription refill and has no other medical complaints. His blood pressure is controlled with his current medication. If at triage his blood pressure was 188/124 and he complained of a headache, then he would meet the criteria for a high-risk situation and be assigned to ESI level 2. If this patient's BP was elevated and the patient had no complaints, he or she would remain an ESI level 5. The blood pressure would be repeated and would most likely not be treated in the ED or treated with PO medications.

"I think he has another ear infection," the mother of an otherwise healthy 2-year-old tells you. "He's pulling on his right ear." The child has a tympanic temperature of 100.2°F and is trying to grab your stethoscope. He has a history of frequent ear infections and is currently taking no medication. He has a normal appetite and urine output, according to the mother.

ESI level 5: No resources. This child has had previous ear infections and is presenting today with the same type of symptoms. He is not ill appearing, and his vital signs are within normal limits. The child requires a physical exam and should be discharged with a prescription.

"I think my son has swimmer's ear. He spends half the day in the pool with his friends, so I am not surprised," the mother of a 10-year-old boy tells you. The child has no complaints except painful, itchy ears. Vital signs: T 97°F, HR 88, RR 18, BP 100/68.

ESI level 5: No resources. This child needs a physical exam. Even if eardrops are administered in the emergency department, this does not count as a resource. The family will be sent home with instructions and a prescription.

A 76-year-old male requests to see a doctor because his toenails are hard. Upon further questioning, the triage nurse ascertains that the patient is unable to cut his own toenails. He denies any breaks in the skin or signs of infection. He has a history of chronic obstructive pulmonary disease and uses several metered-dose inhalers. His vital signs are normal for his age.

ESI level 5: No resources. This elderly gentleman has such brittle toenails that he is no longer able to clip them himself. He requires a brief exam and an outpatient referral to a podiatrist.

"I am so embarrassed!" An 18-year-old tells you that she had unprotected sex last night. "My friend told me to come to the hospital because there is a pill I can take to prevent pregnancy." The patient is healthy, takes no medications, and has no allergies. Vital signs: T 97°F, HR 78, RR 16, BP 118/80.

ESI level 5: No resources. This patient will need a bedside pregnancy test before receiving medication. She may be an ESI level 4, if your institution routinely sends pregnancy tests to the lab.

"Last night I had sex, and we used a condom but it broke. I just don't want to get pregnant," a teary 18-year-old female tells you. Vital signs are within normal limits.

ESI level 5: No resources. This patient will need a bedside pregnancy test prior to receiving medication. She may be an ESI level 4 if your institution routinely sends pregnancy tests to the lab.

"I have an awful toothache right here," a 38-year-old male tells you as he points to his right lower jaw. "I lost my dental insurance, so I haven't seen a dentist for a couple of years." No obvious swelling is noted. Vital signs are within normal limits. Pain 9/10.

ESI level 5: No resources. This patient will require a physical exam. He has no signs and symptoms of an abscess or cellulitis, so he will be referred to a dentist for treatment. In the emergency department, he may be given medications by mouth. On arrival he rates his pain as 9/10, but because he does not meet the criteria for ESI level 2, he would not be given the last open bed.


Related study sets

Educational Psych Exam 2 (Chapters 4 & 5)

View Set

CIS 2050 Final (Chapters 1-9, 11 & 12)

View Set

Pharm review - Cardiovascular, Hematologic, and Lymphatic Systems

View Set

Chapter 30: Abdominal and Genitourinary Injuries

View Set

Chapter 3 Financial Accounting Quiz

View Set

AP U.S. Government & Politics: Chapter 6

View Set

PrepU Chapter 36: Management of Patients with Musculoskeletal Disorders

View Set

chapter 17 Prep U- Implementation

View Set

NURS 212 (Final and last chapter)

View Set